Jump to content

Wikipedia:Reference desk/Science

From Wikipedia, the free encyclopedia

This is an old revision of this page, as edited by 24.150.18.30 (talk) at 04:09, 27 February 2010 (→‎Curling). The present address (URL) is a permanent link to this revision, which may differ significantly from the current revision.

Welcome to the science section
of the Wikipedia reference desk.
Select a section:
Want a faster answer?

Main page: Help searching Wikipedia

   

How can I get my question answered?

  • Select the section of the desk that best fits the general topic of your question (see the navigation column to the right).
  • Post your question to only one section, providing a short header that gives the topic of your question.
  • Type '~~~~' (that is, four tilde characters) at the end – this signs and dates your contribution so we know who wrote what and when.
  • Don't post personal contact information – it will be removed. Any answers will be provided here.
  • Please be as specific as possible, and include all relevant context – the usefulness of answers may depend on the context.
  • Note:
    • We don't answer (and may remove) questions that require medical diagnosis or legal advice.
    • We don't answer requests for opinions, predictions or debate.
    • We don't do your homework for you, though we'll help you past the stuck point.
    • We don't conduct original research or provide a free source of ideas, but we'll help you find information you need.



How do I answer a question?

Main page: Wikipedia:Reference desk/Guidelines

  • The best answers address the question directly, and back up facts with wikilinks and links to sources. Do not edit others' comments and do not give any medical or legal advice.
See also:


February 23

Material in Servo Motor Winding?

Sir/Ma'am

I am looking for the material used in servo motor coil winding. Thanks

And insulator (electrical). Icek (talk) 11:01, 23 February 2010 (UTC)[reply]
All these: enameled copper wire, Solder, Insulator (electrical), see Electric motor#Servo motor Cuddlyable3 (talk) 14:54, 23 February 2010 (UTC)[reply]

Making tea with zingless water

If I boil water in order to make some tea, then get sidetracked for 30 minutes before I've poured it, I have 2 choices:

  • reboil the by now tepid water and then use it
  • start again with a fresh batch of cold water.

But I've always been told NOT to reboil the water, as it's lost its "zing" through the initial boiling. Always start from scratch with new water, is what I've been told to do.

For starters, how true is it about the "zing" thing? If it's true, doesn't that mean that no cup of tea ever has any "zing", because all tea uses boiled water, and boiled water is, according to this theory, zingless?

Assuming that it's true: Does it ever regain its "zing" by somehow reabsorbing oxygen from the atmosphere, and if so, how long does it take? -- Jack of Oz ... speak! ... 10:49, 23 February 2010 (UTC)[reply]

Personally I doubt all this very much. I'm in the habit of filling a kettle first thing in the morning, boiling it, and reboiling it for the subsequent cups of the day, and never notice any difference in the flavour of the tea whatever. Old wives tale if you ask me. --TammyMoet (talk) 11:36, 23 February 2010 (UTC)[reply]
Not that it's pertinent to my question, Tammy, but doesn't your practice mean that you're always boiling more water than you need (until you get down to the last cup/potful in the kettle), and thus wasting both time and electricity? and money? -- Jack of Oz ... speak! ... 20:24, 23 February 2010 (UTC)[reply]

Might be something to do with boiling away impurities? We have quite hard water here with lots of lime in it, and other things, and boiled then cooled water does taste different than water straight out of the tap. Gunrun (talk) 11:59, 23 February 2010 (UTC)[reply]

Lime and other minerals don't get boiled away. They are what's left after the water boils away. Possibly you could boil away the chlorine and byproducts if your water is treated with such. But the usually mentioned difference is oxygen (not that I know if there is any truth to oxygen changing the taste.) Rmhermen (talk) 14:29, 23 February 2010 (UTC)[reply]

Interesting. Health advice for parents of young children drinking baby formula in the UK commonly includes the advice to boil the water once only before using it to make up formula. I've often wondered what it is that they're worried about in reboiling. --Dweller (talk) 12:31, 23 February 2010 (UTC)[reply]

I was told that the more the water boils, the more oxygen it loses, and the flavour of the tea depends on interaction with that oxygen. However, I haven't been able to find a reliable source for that.--Shantavira|feed me 13:47, 23 February 2010 (UTC)[reply]
@Dweller - I think there's either some miscommunication or a bit of social engineering going on. The worry with formula is that parents will boil the formula, which breaks down some of the nutrients. Liquid formula should be heated by placing it in an indirect heat source like a pot of heated water rather than in a microwave or on a stove top. With dried formula, I think the concern is that parents will do up a large batch (properly) and then directly reheat the formula as needed through the day, causing the same problem. If you were to essentially do the operation "in parallel" (mix formula and multiple-times re-heated water in a cup as needed, rather than re-using a pot of re-hydrated formula "in series") there would be no problem. Matt Deres (talk) 15:30, 23 February 2010 (UTC)[reply]
We had these permanent, continuous boiling water urns at our office which just automatically topped the water up as you used it and kept it relatively close to boiling all the time. Never heard anyone complain about lack of zing. They were never turned off, so in the morning you'd be drinking water that has been boiled several times over night. As far as I have seen those kind of urns are not uncommon. If that means anything.. Vespine (talk) 01:11, 24 February 2010 (UTC)[reply]

As far as I know, there are two things one might worry about regarding reboiling of water for drinking. Both has to do with potentially harmful trace amounts of pollutants:

  1. Plain tap water is far from chemically pure. Many of the pollutants in it, have a boiling point higher than the H2O (pure water). Therefore: The longer total time you boil polluted tap water, and the longer total time you keep it steaming hot, then the higher concentration of pollutants you get. (Because some of the water evaporates while pollutants remains in your boiling pot).
  2. Hot water dissolves pollutants far more easily than cold water does, and the longer time hot or boiling water stays in direct contact with a material the more of the material will be dissolved. Something from the plastic or metal (alloy) in your boiling pot is quite likely to give off trace amounts to your water. And that may bee something that you would be better off not swallowing to much of in the long run.

One such cup of tea will not kill you, but some people find it to be worth the effort, in the long run, to make it a habbit not to reboil water for consumption.
For many people, this above explanation is far to tiring. The "Zing"-explanation is much quicker and since many people actually do keep on doing what their knowledgeable (great great) grandmother once stated was the right thing to do, then the zing explanation may be the best one after all.
--Seren-dipper (talk) 04:43, 24 February 2010 (UTC)[reply]

That makes a fair amount of sense, Seren-dipper. -- Jack of Oz ... speak! ... 12:52, 24 February 2010 (UTC)[reply]

I don't have specific scientific evidence but I buy into the theory that brewing good tasting tea requires oxidation, and if the water sits stagnant for too long (such as during successive boilings) it will lose it's oxygen content and the resulting tea will not have as much flavor. Other brewed beverages such as coffee rely on similar processes, however it is up to the raw material (The tea or coffee used) to be susceptible to oxidation during brewing in the first place, so not all teas/coffees will show a change in flavor by changing the oxygen content of the water. --144.191.148.3 (talk) 14:38, 24 February 2010 (UTC)[reply]

I've been unable to find the solubility of oxygen in water at temperatures above 50 degrees C. But I feel that since oxygen redissolves quite quickly in water; and convection currents mixing the surface layes would make equilibrium happen even faster. If someone thinks there is a difference in tea flavour due to oxygen concentration they can put a lid on the cup or put immediately after pouring and see what changes. Polypipe Wrangler (talk) 12:51, 28 February 2010 (UTC)[reply]

Ice on car Windscreens.

Its cold in the UK and I find ice forming on my car windscreen but not on any other windows of my car. I looked at other cars as I was driving to work and this seems to be the case on most of them. Why is the ice only forming on the windscreen and not on the other windows? --Cipher-xs (talk) 11:33, 23 February 2010 (UTC)[reply]

The windscreen usually is the window facing more empty sky than the other windows. The sky serves as an infrared radiation sink. The other windows's loss of heat by radiation is compensated by reception of infrared from the surroundings. Note that this only happens on cloudless, still nights, as clouds radiate back, and wind tends to enhance the heat loss compensation by convection. DVdm (talk) 11:39, 23 February 2010 (UTC)[reply]
Is there not also a precipitation of water from the air, which tends to fall and thus coats glass in proportion to the surface it presents when seen in plan view? --Tagishsimon (talk) 12:26, 23 February 2010 (UTC)[reply]
Probably, yes. Hadn't thought of that one. DVdm (talk) 12:59, 23 February 2010 (UTC)[reply]
There are at least two types of ice commonly called frost. The first is the kind described in the frost article (which itself is two types - convective and radiative), the second is (sadly a redlink) what I have heard called frozen dew. The first is where a surface gets cold enough to freeze water out of the air, the second is where a dew forms and then the temperature drops enough to freeze it. In the first case your windscreen cops the worst of it because it faces the cold sky (as DVdm said). In the second your windscreen cops the worst of it, but rooves, boots (trunks) and bonnets (hoods) also get a pretty white coat --Polysylabic Pseudonym (talk) 08:07, 26 February 2010 (UTC).[reply]

Orbits

I know that most of(Pluto isn't a planet anymore I forgot) the planets in our solar system orbit on the same plain, and orbit in the same direction, because of how our solarsystem was formed or something? Anyway not my question. My question is would this happen naturally anyway? My friend says that if we were to introduce a new planet to the system it would eventually level into the same plane as the other planets, regardless of where you placed it (Obviously giving it enough velocity to orbit the sun), whereas I can't see why it would. Why couldn't a planet orbit the sun at a 90 degree angle to everything else. Also is there any reason why a planet couldn't orbit in the opposite direction of the other planets? Gunrun (talk) 11:57, 23 February 2010 (UTC)[reply]

He might be thinking of Accretion disc where a circular cloud form into a pancake-flat disk by conservation of angular momentum when some material fall into the center and become the star. This explains why all "original" planets orbit in the same direction, they were formed from the same pancake.
I can't see how what your friend proposes could happen for a new planet at a steep angle to the rest. But the conservation of angular momentum means that if the new planet gradually "falls into place", it aquires angular momentum in the solar system and therefore other planets must loose it. At the same time, the total kinetic energy must be conserved too. I can't see how this should happen, so I'm guessing your friend is wrong. :-/ —Preceding unsigned comment added by EverGreg (talkcontribs) 12:50, 23 February 2010 (UTC)[reply]
Yeah - I think it would happen, although it would take a very long time - and Pluto has not yet done that...further evidence that it was not formed like the other large bodies in the solar system. Let's imagine an object orbiting in a plane that's (say) 30 degrees from the ecliptic - for half of each orbit it's on one side of the ecliptic, for the other half, it's on the other side. The gravitational fields of every single other body in the solar system are exerting a force on that object. The vector sum of all of those forces is overwhelmingly towards the sun (which is why the object is orbitting) - but the other planets are each creating an additional very tiny pull - which can be resolved into a net force parallel to the ecliptic and another at right angles to the ecliptic. The 'parallel' force will vary as the various planets, moons and other debris go around their orbits and seem more or less random (although dominated by wherever Jupiter happens to be). But the force at right angles to the ecliptic is ALWAYS in the direction of the ecliptic. When the object is below the ecliptic, this net force is always in the upwards direction, when it's above the ecliptic, the force is down. This means that the orbit is going to gradually be pulled around until it's in the same plane as everything else. You can see this kind of effect happening in (for example) Saturns rings. When two house-sized chunks of ice smack into each other, the fragments will scatter off in all directions - if it were not for this kind of effect, the rings would quickly break up and turn into a random spherical shell of chaotic colliding debris - but their relative gravitation forces them all into an almost perfectly flat, insanely thin disk. That's likely why galaxies are pretty flat too. SteveBaker (talk) 13:50, 23 February 2010 (UTC)[reply]
When the planet is attracted to the ecliptic, it seems that the planet would oscillate around the ecliptic, not settling into it. The sum of kinetic and potential energy for the new planet would have to be conserved after all. EverGreg (talk) 14:08, 23 February 2010 (UTC)[reply]
Well, conservation is managed because the other planets (etc) get pulled slightly towards the plane of this rogue body - so the entire plane of the ecliptic tilts slightly in reaction to this new body. However, unless the new body is utterly gigantic, that effect would probably be too small to measure. SteveBaker (talk) 21:21, 23 February 2010 (UTC)[reply]

Thanks steve,think that answers my first question pretty well. Does this mean if you had a planet orbiting in the other direction to the others (say they go clockwise this one goes counterclockwise) it would eventually get slowed down and thus move closer to the sun (Like how a space station de-orbits) presumably bringing it onto a collision course with a planet coming the other direction, or would such a planet be impossible?) Gunrun (talk) 13:57, 23 February 2010 (UTC)[reply]

Yes to this new hypothetical question that doesn't explain where the rogue planet came from. Eventually all the established planets would orbit very slightly slower but we can't predict whether the rogue planet collides with a planet before it crashes and burns in the Sun. Cuddlyable3 (talk) 14:40, 23 February 2010 (UTC)[reply]
Didn't you mean "would orbit very slightly more slowly", Cuddly? (Sorry, couldn't resist).  :) -- Jack of Oz ... speak! ... 20:35, 23 February 2010 (UTC)[reply]
So in an act of pure malice after being downgraded to a mere dwarf planet, Pluto wreaks revenge by smacking into the Earth in a few billion years? SteveBaker (talk) 21:24, 23 February 2010 (UTC)[reply]
hey, wouldn't you? heaven hath no fury like a planet scorned...--Ludwigs2 21:29, 23 February 2010 (UTC)[reply]
...a dwarf planet scorned... :-) SteveBaker (talk) 04:05, 25 February 2010 (UTC)[reply]
If it would just ditch Charon, It would be back to planet status. Googlemeister (talk) 21:42, 23 February 2010 (UTC)[reply]

freezing point

What is the freezing point of 80 proof alcoholic beverages?

Thank you.

Ken Carter

—Preceding unsigned comment added by 97.91.189.14 (talk) 15:28, 23 February 2010 (UTC)[reply] 

Sorry I can't help but I removed your email, in accordance with our guidelines and to avoid you getting spam. --TammyMoet (talk) 15:39, 23 February 2010 (UTC)[reply]

This site suggests -23 °C (-10 °F) for a 40% ethanol solution (which equals 80 proof); however this site suggests -33C for 84 proof. Vodka is essentially diluted ethanol, so it should freeze at that temperature, as will most other spirits. Other alcoholic beverages (like Southern Comfort) may contain significant amount of sugars that may affect the freezing temperature. Regardless, it won't freeze in a household freezer. -- Flyguy649 talk 16:53, 23 February 2010 (UTC)[reply]
One thing I have heard (and this is unsourced), is that if you start to freeze vodka, small ice crystals will begin to form. These crystals are mostly water, so if you remove them, you increase the proof. Googlemeister (talk) 17:43, 23 February 2010 (UTC)[reply]
See Fractional freezing. -- Flyguy649 talk 18:04, 23 February 2010 (UTC)[reply]

liver

pls can you help me to answer this question how can diesel affect the liver of a male wistar pls this is a project work —Preceding unsigned comment added by 41.204.224.39 (talk) 15:48, 23 February 2010 (UTC)[reply]

First have a look at the effects of diesel on wistar rats in general. Then as a followup, take a look at this research report, for example, which discusses other systemic effects, including the liver. You might be interested to know that Google Scholar is available to help you find specific scientific publications. I just searched for "wistar diesel liver" and found a huge quantity of interesting results. If you know what you're looking for specifically, you can narrow down the search. Nimur (talk) 16:43, 23 February 2010 (UTC)[reply]

Octanediol

What is it and what are its uses? --Reticuli88 (talk) 16:04, 23 February 2010 (UTC)[reply]

oh, is the same thing as Octane-1,8-diol? --Reticuli88 (talk) 16:05, 23 February 2010 (UTC)[reply]

"Octanediol" is a somewhat generic name. It probably has an implied meaning of one specific chemical (or group of them) in some contexts. The "1,8" is one specific chemical of this type ("diol" is "two OH groups", the "1,8" says there is one of them on each end of the chain rather than scattered along the middle). DMacks (talk) 16:34, 23 February 2010 (UTC)[reply]

Do you know which brands contain Octane-1,8-diol as descibed in the article? --Reticuli88 (talk) 16:38, 23 February 2010 (UTC)[reply]

Fisher Scientific sells many types, each specified with a full IUPAC name. They do carry 1,8-octanediol. Nimur (talk) 16:47, 23 February 2010 (UTC)[reply]

LOGIC GATES

Q1 Explain factors to be considered when constructing logic gates? Q2 Explain the positive as well as negative logic? Q3 Explain Digital Families and their characteristics? —Preceding unsigned comment added by Jyoti.prakash1507 (talkcontribs) 17:46, 23 February 2010 (UTC)[reply]

Read the article Logic gate. I have a feeling that you will also find Binary numeral system interesting. However we shall not do your homework for you. Cuddlyable3 (talk) 18:19, 23 February 2010 (UTC)[reply]

Agni III

Why does not the Agni III burn when it reenter in the atmoshphere. —Preceding unsigned comment added by 58.68.8.189 (talk) 17:49, 23 February 2010 (UTC)[reply]

I'm not sure about the Agni III in particular, but our atmospheric reentry article discusses a variety of techniques for engineering vehicles to survive reentry. -- Coneslayer (talk) 17:54, 23 February 2010 (UTC)[reply]

time dilation?

Does time really move fast near to gravitational field and slow when at far from gravitational field? I feel that though time is considered as the fourth dimension but still it is virtual in concept. So when we say that time moves fast or slow i think we take into account relative time (with respect to something) and not the absolute time. Though near Black hole time change is certain but still the absolute time(Universal Clock) has the same time. —Preceding unsigned comment added by Itsrohit (talkcontribs) 18:12, 23 February 2010 (UTC)[reply]

Yes. Time dilation is a real effect.
It has been verified with atomic clocks. See Time_dilation#Experimental_confirmation.
See also Effects_of_relativity_on_GPS
APL (talk) 18:58, 23 February 2010 (UTC)[reply]
Oh. To answer your second question : No, as far as we know there is no "universal time". Time itself runs differently for observers in different reference frames. There can be no "universal time" without an absolute reference frame, but there isn't one. See Special_relativity#Lack_of_an_absolute_reference_frame.
APL (talk) 19:02, 23 February 2010 (UTC)[reply]
(ec)Yes, the effect can be measured with real clocks, which of course don't care about theories or dimensions. Also note that there nothing virtual about time. Time is just what we read on clocks, which is the reason why a concept like absolute time is as meaningless as a concept like, say, blue time. DVdm (talk) 19:06, 23 February 2010 (UTC)[reply]
Time is a completely variable thing. Because it's affected by both the speed you're moving and the force of gravity, as I type this email, the tips of my fingers are experiencing a very slightly different rate of time flow than the rest of my body because they are moving relative to it. Time can be running at different rates at every point in the universe. It's even more weird than that. The entire concept of two events "happening at the same time" is utterly meaningless unless they are at the exact same point in space - and so is the concept of "Event A happened before Event B". We can only use those relationships because we are all moving at very close to the same speed and in roughly the same gravitational field - and the differences in the rate of flow of time on "human scales" is so microscopically small that we can't tell that we have this rather deep misunderstanding of the way that the universe operates. Even when we understand this intellectually - the "gut feel" that time stays very firmly nailed down is hard to shake. SteveBaker (talk) 20:51, 23 February 2010 (UTC)[reply]
"The future is already here - it is just unevenly distributed" - William Gibson -- Finlay McWalterTalk 21:00, 23 February 2010 (UTC)[reply]
Don't go too far with the weirdness! "A before B" (or the reverse, as appropriate) is a perfectly valid, invariant relation for all events with timelike separation, which is quite the superset of "at the exact same point in space". You get that some observers will say (in their coordinates) "A was before B" and others "B was before A" when the separation is spacelike. --Tardis (talk) 21:21, 23 February 2010 (UTC)[reply]
Yeah - you're right that in a lot of cases it's OK to say that two events happened in a particular order - but not in all cases. Basically if the two events happen so close together time - but so far in space that light from one event has not yet had time to reach the place where the other event happens before it happens - then some observers may disagree about the order they happened in. If there is time for light to travel between them then their order is well-determined for all observers. SteveBaker (talk) 21:39, 23 February 2010 (UTC)[reply]
One of Einstein's great observations—and most controversial statements, even in his own time—was that there is no way to possibly measure absolute time and thus it doesn't actually exist. (Or, put another way, it must thus be an inherently metaphysical concept, and not part of science.) He believed in local time—as did many other people at the time—but he was really the only one who threw out the concept of absolute or universal time as being superfluous and unwarranted. Over time scientists have come to go with Einstein's approach—simply because his definition of time does seem to work out, does produce the weird experimental effects he predicted it would, and does not seem to require an idea of absolute time to work. --Mr.98 (talk) 22:11, 23 February 2010 (UTC)[reply]

Do most people get enough vitamins and nutrients from their regular diet?

I've always been slightly suspicious of dietary supplements like multivitamins (I believe Sheldon from The Big Bang Theory once described them as things that "make your urine very expensive", which I thought was apt), so I figured that once and for all I'd read the damn wikipedia article and the arguments for and against it. The arguments against them in the article are a lot stronger than the arguments for them unless you're a post-menopausal woman or suffering from malnutrition (in which case you have bigger problems, I suppose). The pro-side is basically one paper, one editorial and one doctor, while the con-side is the NHS, HHS and the UK Food Standards Agency.

However, there was one argument that wasn't addressed that I'm very interested in: doesn't most people living in affluent countries pretty much get everything they need (in terms of vitamins and minerals and such) just from a regular diet, even a pretty unhealthy one? I mean, comparatively speaking, isn't a western diet is hugely diverse by historical standards? Look at a Big Mac: not the healthiest of meals, but it has salad, tomatoes, beef, bread, cheese and pickles in it. I mean, that's like an explosion in different varieties of nutrients! Compare that to a stone-age meal consisting of badly cooked mammoth and a few berries, and it's hard to see the case for modern people needing additional nutritional supplements.

Is my intuition correct? Surely there has to have been studies that look at the average persons diet and checked whether it matches up with the Recommended Daily Intake? Belisarius (talk) 18:28, 23 February 2010 (UTC)[reply]

I've heard the same thing, and I'd love to have a definitive understanding of how often people do or don't have aspects of malnutrition while on a typically Western diet. I certainly know of anecdotal cases of people in the modern world where supplements were important, but those generally involve restricted diets that wouldn't necessarily be called typical. For examples, vegans often take supplements to deal with nutrient that can be hard to obtain from an all plant diet. My mother does similar things to deal with restrictions caused by her food allergies. And I once knew a grad student that was diagnosed with scurvy (lack of vitamin C). It is very rare to see that kind of basic malnutrition in the USA, but it is still evidently possible to make such poor diet choices that one unwittingly misses out on basic nutrients. So, I guess my point is that supplements do have uses, but I don't know how useful or not they are for the typical Westerner. Dragons flight (talk) 19:00, 23 February 2010 (UTC)[reply]
Short answer: yes they do get enough vitamins. If they didn't they would be dead. But since 100% of people are alive, then by logical analysis, everyone gets enough vitamins. Maybe not -ideal-, but they seem to get by alright. Besides, a severe deficiency in any important materiel will result in particular cravings. Vranak (talk) 19:04, 23 February 2010 (UTC)[reply]
A lot of things have their effects over long periods of time and shorten your life. By your logic, smoking is perfectly safe. 92.29.57.43 (talk) 23:52, 23 February 2010 (UTC)[reply]
What a ridiculous answer. They aren't dead yet, so obviously everyone is in good health. That's just stupid. Also, the body may be smart about some things, but it isn't magic. It can only give you cravings for things it is internally set up to measure and understand, which isn't everything. Most Westerners do fine, but don't mistake that for saying proper nutrition isn't relevant to good quality of life. Worldwide, malnutrition still contributes to ~60% all deaths every year, mostly in the impoverished third world countries. Dragons flight (talk) 20:13, 23 February 2010 (UTC)[reply]
It's not the least bit stupid. Dead is dead is dead. But yes, your point is well-taken. I generally reserve my consideration for countries that don't suffer widespread famine. In fact I would submit that any nation where people starve to death is no nation at all. Vranak (talk) 20:40, 23 February 2010 (UTC)[reply]
Sure! Like Ireland in the mid-19th Century, or China during the Great Leap Forward. Ireland and China weren't nations! They were, um, these things ... where people of similar ethnic groups governed by the same people, um ... just happened to live ... But definitely not "nations."63.17.65.39 (talk) 02:20, 24 February 2010 (UTC)[reply]
It is kind of stupid. Malnutrition is not the same thing as starvation. Plenty of people are malnourished in all nations. It is not an issue of "widespread famine"—there are plenty of places where a poor diet is common amongst huge sectors of the population, and the results of that are medically palpable (lowered IQs, shorter statures, etc.). Heck, even if you are extremely well-fed you can easily run afoul of malnutrition if you don't eat the right thing. I know of a guy who ate nothing but hot dogs and soda for an entire summer and got scurvy as a result. (Which was itself pretty stupid, but it illustrates the point well!) --Mr.98 (talk) 22:14, 23 February 2010 (UTC)[reply]
I know some purists will object, but you can prevent scurvy while on the hot dog diet simply by adding ketchup. Googlemeister (talk) 14:06, 24 February 2010 (UTC)[reply]
Alright, I see we are not on the same wavelength here. Vranak (talk) 01:03, 24 February 2010 (UTC) [reply]
Bluntly, Mr.98 is on the right one, and you're on the wrong one. "Not getting enough nutrients" is not remotely close to the same thing as "dead".
Just as an example, if I recall correctly, having inadequate serum levels of vitamin D means you're more likely to get cancer, or at least some kinds of cancer. Does everyone who doesn't get enough D, get cancer? Certainly not. But that doesn't mean you can be sanguine about D deficiency, even if you haven't yet gotten cancer.
Of course you don't want too much D either; it's a complicated question. --Trovatore (talk) 01:09, 24 February 2010 (UTC)[reply]
The only proof that someone did not definitively get enough nutrition is if they die. Otherwise they can make a recovery. So there is no absolute way of calling someone 'malnourished' because they may just pull through, and their deficiency was never really that critical, if it did not quite finish them off. Still, your objection is noted and respected. Vranak (talk) 02:01, 25 February 2010 (UTC)[reply]
So you're saying you don't mind getting cancer, heart disease, or high blood pressure from incorrect nutrition, provided there's no actual proof that that was the cause? --Trovatore (talk) 02:07, 25 February 2010 (UTC)[reply]
It has become abundantly clear to me that you lads are not interested in the truth, so I take my leave from this debate. Vranak (talk) 03:58, 25 February 2010 (UTC)[reply]
It's become abundantly clear that you're ignoring good answers. If someone has a deficiency of a certain vitamin, they become diseased. Disease does not equal death, it means that you're not at optimum efficiency and if you continue to degrade in health, you could die. Furthermore, there are not just two levels of well-being when it comes to vitamins. For example, someone who has a lack of vitamin C will get scurvy, but it doesn't mean they'll die from it. It means they'll become ill, and if they continually lack the vitamin in sufficient quantities over time, they'll die. However, if they get treatment, they'll live. If they get just under the bare minimum quantity of vitamin C, they can present with some symptoms of scurvy but not die. There's the proof that your theory is flawed. I can prove that the last person mentioned didn't get enough vitamin C because they presented with the symptoms, but they didn't die. Therefore, you can prove someone has a lack of nutrients even if they're not dead. Regards, --—Cyclonenim | Chat  18:14, 25 February 2010 (UTC)[reply]
You can see what percentage of the most commonly needed stuff is in your food because it's right there on the mandatory labelling - so you don't have to take our word for it...you can figure it out for yourself. But even the most dull and non-nutritious seeming breakfast cereal has something close to the recommended amount of just about everything (and that's not counting the milk you pour onto it - which, remember is sufficiently "well rounded" in nutrients to grow a tiny calf into a modest sized cow with no supplementation). Since that's only the first meal of the day - it's pretty clear that you'd have to go out of your way to pick a sufficiently poor diet. The exceptions (and they are important ones) are pregnant women (who will reduce the risk to the baby by taking some specialised supplements), nursing mothers, older people, people with dark skins who live in the extreme north or south of the planet, sick people, malnourished people and so forth. But if you are in your prime and eating "normally" - you should be just fine without any kinds of special supplements whatever. Like I said...check the product labelling and then you'll know for sure. SteveBaker (talk) 20:26, 23 February 2010 (UTC)[reply]
Well, the labeling is only for a fairly restricted group of nutrients — by no means everything that you might want more of (or, sometimes, less of). Most phytochemicals that aren't actual recognized vitamins, for example, don't get called out on the label, but your health is at more risk if you don't get enough of them. I'm talking about things like soluble fiber, polyphenols, many many more. Omega-3 fatty acids are another group of nutrients that many Westerners don't get enough of. You don't have to be poor to be in this boat; it might be that you just don't particularly enjoy the foods that provide these nutrients. The best solution to that is to recalibrate your food preferences, which is possible to some extent. But if you absolutely can't, or refuse to, then supplementation may be an option. --Trovatore (talk) 22:34, 23 February 2010 (UTC)[reply]
"Westerners"? If that means you consider Europe, Aus, NZ, etc to be homogenous with the USA then that is a mistake - we tend to eat different foods and have a different culture and conventions. 92.29.57.43 (talk) 23:47, 23 February 2010 (UTC)[reply]
You're right, it was a careless choice of words. I meant people in affluent countries, in countries where access to food isn't really a problem. Belisarius (talk) 00:42, 24 February 2010 (UTC)[reply]
I read a detailed academic study online a while ago that said that taking a multivitamin everyday was worse for you than not taking one. Because of that, I stopped taking them and concentrated on eating more vegetables, fruit, and oily fish. Have not been able to find a link to that article unfortunately. 92.29.57.43 (talk) 23:51, 23 February 2010 (UTC)[reply]
Vitamin B12 deficiency can be a real problem for vegans.--Shantavira|feed me 06:53, 24 February 2010 (UTC)[reply]
Despite deciding to stop taking a daily multivitamin after reading a scientific paper about this, I do take Vitamin D during the winter, after reading a Scientific American article about this that I think was published in November 2008. The modern diet has a lot of saturated fat and salt, both of which I am trying to avoid by avoiding processed food, and also red meat and dairy products. 92.29.32.229 (talk) 20:57, 26 February 2010 (UTC)[reply]

In the reference to Linus Pauling it states that he died in 1994 and that he taught at Oregon State University in 1996. Must have been interesting classes. I don't know if he actually taught at OSU but I think he didn't do it in 1996.

Regards all,

Stephen Wilson —Preceding unsigned comment added by Zashiban (talkcontribs) 19:01, 23 February 2010 (UTC)[reply]

I don't see where it says that he taught at OSU. It says that the Linus Pauling Institute moved there in 1996, but that's a different thing. Looie496 (talk) 20:16, 23 February 2010 (UTC)[reply]
Err, yeah, read a little closer. It does not say what you think it does. --Mr.98 (talk) 21:33, 23 February 2010 (UTC)[reply]
Mr 98 it really would be way more helpful if you could say exactly where the problem is. If you have spotted the discrepancy, (and well done for that) let's get it sorted and move on. Caesar's Daddy (talk) 08:43, 24 February 2010 (UTC)[reply]
This is best handled on the belonging talk page. However, my curiousity was spiked, and I felt the need to quote what has been written (and not edited, by the look of the history): In 1996, the Linus Pauling Institute moved from Palo Alto, California, to Corvallis, Oregon, to become part of Oregon State University, where it continues... Bolding is mine. All other searches for "1996" yield results outside this context. However, he studied at OSU (first line in #Career). I can conclusively say that Mr 98 is wrong. 77.18.74.19 (talk) 10:06, 24 February 2010 (UTC)[reply]
Ummm...I don't think Mr98 said anything wrong...perhaps you were confusing him with the OP? It looks like Mr98 made basically the same response as the rest of you.PhySusie (talk) 17:28, 24 February 2010 (UTC)[reply]
Ah, I thought Mr98 was replying to Looie496. :) My bad. 77.18.12.14 (talk) 16:55, 25 February 2010 (UTC)[reply]
In theory, posts that are indented are responding to the post which is of lesser indent to the one above it. If I were responding to Looie496, I would have indented it one more time. (I'm seriously surprised that this is any real confusion.) --Mr.98 (talk) 14:19, 26 February 2010 (UTC)[reply]

Sodium

How can I extract pure sodium from baking soda (sodium bicarbonate)? --J4\/4 <talk> 20:11, 23 February 2010 (UTC)[reply]

Basically you can't. See Downs cell for the simplest method of producing pure sodium. Looie496 (talk) 20:21, 23 February 2010 (UTC)[reply]
Why not? Baking soda has sodium in it, after all. --J4\/4 <talk> 20:27, 23 February 2010 (UTC)[reply]
I didn't mean that it's impossible in principle, just that it's impossible without very serious equipment. The article I pointed to should give you an idea of why. I don't actually know how to do it with sodium bicarbonate, but it must be harder than the Downs cell process. Looie496 (talk) 20:31, 23 February 2010 (UTC)[reply]
In layman's terms: Sodium (as I'm sure you know) is highly reactive. When it reacts with things it puts out a LOT of energy. Hence, to get it separated back out again, you have to somehow replace all of that energy and fight Sodium's tendency to react with whatever happens to be nearby. It follows that getting pure sodium out of its compounds is difficult. (You can't get it out of common table salt either!) SteveBaker (talk) 21:31, 23 February 2010 (UTC)[reply]
(e/c) put simply, sodium is very reactive, you can ionize it in solution fairly easily (plop plop fizz fizz) but getting it out of solution without it recombining with the other solutes (or other materials) is a bear and a half. --Ludwigs2 21:34, 23 February 2010 (UTC)[reply]
IMHO, if you want detailed advice on how to make sodium, you'll be better off using table salt as your starting material. The best method is to melt it (it melts at ~800 C) and then electrolyse it using a graphite anode and a steel cathode with some kind of hood over it to catch the molten sodium as it rises and thus prevent it from reacting with air. Warning: chlorine gas given off at the anode is highly toxic, avoid inhalation. Also, all solid/liquid materials will be red hot, don't burn yourself. As for the other, common-sense advice: Don't try this at home, at least unless you really know what you're doing. Clear skies to you 24.23.197.43 (talk) 01:27, 24 February 2010 (UTC)[reply]
Let's try also to explain this in a different manner. Sodium comes in two forms: Sodium metal, where each atom has 11 electrons, and ionic sodium, where each atom has 10 electrons. The nucleus of sodium contains 11 protons, so that makes sodium metal neutral, and sodium ions have a charge of +1. Energetically, it turns out that sodium ions are MUCH MUCH MUCH more stable than neutral sodium. That's why when you add sodium to water, it releases so much energy; in giving an electron to the water, the sodium becomes more stable, and in the process releases the pent-up energy that was making it unstable (which explains the big boom). You can look at it this way:
  • Na0 --> Na+1 + 1 electron + a shitload of energy
Well, what the OP is proposing is to reverse the reaction; take the IONIC form of sodium (which is the form in sodium bicarbonate) and turn it into the METALIC form of sodium (the neutral one). So if you run the reaction backwards; you would get
  • Na+1 + 1 electron + a shitload of energy --> Na0
So the first thing you need is a shitload of energy; basically you need exactly as much (or more) as you release if you were to add the pure sodium metal to water. As the OP is aware, that's quite a bit. The second problem is that you need to do it in such a way as to keep the sodium metal away from anything that would give it a pathway to give away that electron again. Any environment that contains any water, for example, will just lead to your newly-made sodium metal instantly reverting back to its ionic form, and you'll never get a measureable amount of the pure metal. The Downs cell noted above is the only really practical method of creating metallic sodium, and it is a complex process that requires some exotic equipment and a lot of energy to work. --Jayron32 16:40, 24 February 2010 (UTC)[reply]
Water is partly ionic, so it has free H+, and OH- ions. Table salt is Na+ Cl-. When you electrolyse a solution, the ions with with least electrode potential with be released. H+ is less than Na+. This is why electrolysis of sodium salts is normally done with molten salt, as is stated above. CS Miller (talk) 18:57, 25 February 2010 (UTC)[reply]

acid

once spilled strong acid has dried is it inert or is it still caustic —Preceding unsigned comment added by 67.246.254.35 (talk) 22:15, 23 February 2010 (UTC)[reply]

AFAIK it depends on the volatility of the respective acid anhydride. Hydrochloric or nitric acid may volatilize completely, leaving no caustic residue (unless it had soaked into the surface instead of evaporating); sulfuric or hydrofluoric acid, on the other hand, will remain caustic after having dried up. However, even with fairly volatile acids, there's no way to really be sure that it has really dried up and not just soaked into the surface. Always clean up acid spills promptly and follow safety guidelines. 24.23.197.43 (talk) 01:35, 24 February 2010 (UTC)[reply]
"Acids" are usually solutions of acidic chemical dissolved in a solvent. So you have to figure out what you mean...is the solvent evaporating, leaving an extremely concentrated acidic residue, or is the acid chemical itself evaporating. Most of the common strong acids, and many of the common weaker ones (that still might be very corrosive) are volatile enough to evaporate completely with their solvent: if you are in a situation where the water is evaporating, the acid will also. To put another way, acids are acidic--evaporating them merely changes the physical state of them, so whereever that physical acid chemical goes, in whatever form it is, that's where the acidic properties exist. DMacks (talk) 02:52, 24 February 2010 (UTC)[reply]


im talking about car battery acid ( i think sulfuric ) —Preceding unsigned comment added by 67.246.254.35 (talk) 03:21, 24 February 2010 (UTC)[reply]

Yes, car battery acid is sulfuric acid. Also, what kind of surface did you spill it on? If you spilled it on dirt, grass, or sand, it probably would have soaked in and would not leave any caustic residue. If you spill it on concrete, tile, or wood, I think the acid would evaporate with the water. So in any case, I think it would be safe, depending on how long ago you spilled it. --The High Fin Sperm Whale 04:12, 24 February 2010 (UTC)[reply]
You can neutralise it with sodium bicarbonate which will fizz as it reacts. There may be some bisulfates left after sulfuric acid reacts with concrete, and these can be neutralised. Graeme Bartlett (talk) 11:25, 24 February 2010 (UTC)[reply]


the battery exploded in my car hood the acid is all over the engine parts ect under the hood —Preceding unsigned comment added by Thekiller35789 (talkcontribs) 15:25, 24 February 2010 (UTC)[reply]

In that case, you should take your car to a mechanic to check for acid corrosion to engine parts and bodywork -- most car parts are made of carbon steel, and sulfuric acid will corrode it readily and could cause serious damage. (BTW, why don't you sign your posts?) 24.23.197.43 (talk) 05:50, 28 February 2010 (UTC)[reply]

"local hour angle" in astronomy

I'm trying to do some astronomical calculations based on the "Astronomical Formulae..." book by Jean Meeus. I almost have it, but I have a question about "local hour angle". On page 44 (chapter 8) he defines (along with the observer's latitude) "H = the local hour angle, measured westward from the South". Is the local hour angle simply the observer's degrees of longtitude? Bubba73 (You talkin' to me?), 22:42, 23 February 2010 (UTC)[reply]

I'm not certain of this, but I think it refers (in the context of your probable interest) to the angle in right ascension between the meridian and the horizon, measured from south to west. If RA is in degrees, divide by 15 to put it into hour:minute format. This is also called the western diurnal semi-arc. More broadly (outside your context), it is "the angle between the great circle that passes through the poles, and that which bisects a specific point in the heavens" ("Encyclopedia of Astrology" by Nicholas Devore). 63.17.65.39 (talk) 02:35, 24 February 2010 (UTC)[reply]


The local hour angle is the angle measured westward from the meridian (that is, the projection of the line of longitude the observer is standing on onto the celestial sphere) to the hour circle of the celestial object, measured westward along the celestial equator. See "hour angle" in the Glossary of the Astronomical Almanac. I don't really see how thinking about "south" helps understand this. Jc3s5h (talk) 02:53, 24 February 2010 (UTC)[reply]
Yes, the mention of "south" confuses me. I think it may be an error in the book, since the same phrase is used for azimuth (where it does make sense). Bubba73 (You talkin' to me?), 03:45, 24 February 2010 (UTC)[reply]

Perhaps I need to back up some. I am writing a program to calculate the altitude and azimuth of the Moon, given (my) latitude, longitude, and the time. Chapter 30 of the book by Meeus give how to calculate the geocentric latitude and longitude (lambda and beta). The program is doing that - it agrees with the example in the book. (The Moon's motion is complex.) But I'm stuck on getting from there to altitude/azimuth.

Chapter 8 of the book shows how to calculate alt/az from my latitude, the declination, and the local hour angle. The description of the local hour angle isn't clear to me. It shows two ways to calculate the hour angle, based on the local sidereal time and the right ascension (alpha). But I don't know if these r.a. and dec correspond to the geocentric lat and long.

Just before that it shows how to calculate right ascension and declination from lambda, beta, and the tilt of the Earth's axis. I suppose I need to do that to get the declination to use in the alt/azimuth calculation, but I'm not sure.

So my problem is how to get from geocentric lat/long to alt/azimuth, given my latitude, longitude, and the time. How is that done? (I'm confused.) Bubba73 (You talkin' to me?), 05:01, 24 February 2010 (UTC)[reply]

Forget that "south" business. It is conflating hour angle with something else . If someone was considering a point on the celestial sphere, say the position of a star, and you wished to instruct him how to find another point, you would have to tell him in what direction to look from there. Due South is 0°, Southwest is 45°, East is 270°, and so forth. Also supply the distance, and you've done it. B00P (talk) 06:39, 24 February 2010 (UTC)[reply]
Don't forget the south, it's correct and belongs in the explanation (assuming you're in the northern hemisphere). If you stand facing south, an object on the meridian, in front of you, has an hour angle of 0h. Now turn around and face north. An object directly in front of you has an hour angle of 0h if it's above the north star, but an hour angle of 12h if it's below the north star. To put it another way, each day and night a circumpolar star will cross the meridian twice, but it's the meridian to the south that it crosses at 0h hour angle. -- Coneslayer (talk) 12:28, 24 February 2010 (UTC)[reply]
That said, the formulas should "just work" for your calculation, and you don't need to worry that HA is measured from the south. You do, of course, need to worry about whether azimuth is measured from the north or south. -- Coneslayer (talk) 13:10, 24 February 2010 (UTC)[reply]
OK, I have Astronomical Algorithms, which I think is an older edition of the book, but unfortunately the chapter/equation numbers don't match up. I think you've basically got it figured out. You have geocentric latitude and longitude (lambda and beta). Convert those to RA & dec (alpha and delta) using the relations you've found. Then get the hour angle based on RA and local sidereal time (or RA, observer's longitude, and Greenwich sidereal time), and use the relations you've found to convert HA & dec to altitude and azimuth. All good? -- Coneslayer (talk) 13:03, 24 February 2010 (UTC)[reply]
Astronomical Algorithms is actually a newer book (also by Jean Meeus) than the one I have. I'll try to finish it up, based on what seems right. Unfortunately, there is no complete example of getting the Moon's position in alt/azimuth in the book, so I'll have to check it some other way. Bubba73 (You talkin' to me?), 01:40, 25 February 2010 (UTC)[reply]
Bubba, I have the correct answer (and my answer above from "63.17.65.39" was correct, but I thought you wanted it relative to the horizon -- but I see you're referring to the moon). It's very simple: subtract the right ascension of the moon from the right ascension of the meridian (RAMC). RAMC is the same as local sidereal time. If the RAMC is less than the RA of the moon, add 360 degrees (or 24 hours) to the RAMC. So it's just subtraction! 63.17.57.56 (talk) 02:21, 25 February 2010 (UTC)[reply]
Another way to think of "local hour angle" -- in this case, the LHA of the sun -- is "At my latitude on earth, how many 'time zones' away is the sun at 'high noon'?" So if you're in New York and it's 3 PM, it's noon on the west coast -- so the "hour angle" of the sun in NY is approximately 3:00, because LA is 3 time zones from NY. (A better example would be two cities at the same latitude, but you get the point.) The same goes for the moon or anything else, except it's "the moon's noon," i.e. high point. Along the same line of geographical latitude, the hour angle is simply how far away in geographical longitude (equivalent to right ascension) a point in the sky has reached its maximum height ("due south"). If the moon is due south (at its high point) in LA (longitude 120ish), the moon's hour angle in NY (longitude 75ish) will be appproximately 45 degress, or 3 hours. (Again, literally speaking, it has to be along the same line of latitude on earth, and time would be measured as on a sundial, not a regularized clock. I just say this to avoid the usual pedantic corrections.) 63.17.57.56 (talk) 04:09, 25 February 2010 (UTC)[reply]
It is my bedtime - I'll read that tomorrow. But in the meantime there are two things I don't understand. The final step is calculating the altitude and azimuth. Meeus shows how to do that in equations 8.5 and 8.6. But these equations depend only on the observer's latitude, hour angle, and delta (the declination). Why doesn't the R.A. enter into it? Secondly, is that declination the parameter calculated for the Moon in geocentric coordinates? Bubba73 (You talkin' to me?), 06:16, 25 February 2010 (UTC)[reply]
You needed the RA to get the hour angle, so that's how RA enters into it. The declination (along with the RA) is calculated from the geocentric coordinates (you originally wrote: Just before that it shows how to calculate right ascension and declination from lambda, beta, and the tilt of the Earth's axis. That's where the declination comes from.) -- Coneslayer (talk) 12:20, 25 February 2010 (UTC)[reply]


February 24

Traffic fumes versus tobacco smoke

How does the deadliness of traffic fumes compare with those of tobacco smoke? Would living a few feet away from very busy roads - unfortunately not uncommon in the UK - be equivalent in its health risks to smoking X number of cigarettes a day? The (very) microscopic particles from diesels are one serious risk for example. 92.29.57.43 (talk) 00:51, 24 February 2010 (UTC)[reply]

Prob'ly not -- when you smoke cigarettes, you deliberately inhale the smoke, maximizing your exposure to all the nasty stuff in it. As opposed to just breathing air that happens to contain a certain amount of smoke, which is what you do when you live next to a busy road. Plus, the natural air currents tend to dilute the traffic fumes to a very low concentration, unlike when you smoke tobacco. So no, living next to a busy road is nowhere near equivalent to smoking X cigarettes a day -- more like sitting in a bar where other people smoke. But if you want a gram-for-gram comparison of engine exhaust vs. tobacco smoke, diesel exhaust contains more aromatics and heavy metals, while cigarette smoke contains more of the nasty carbonyl compounds (aldehydes/ketones) and amines/nitro compounds (but some aromatics and heavy metals as well). So gram-for-gram, I'd say it's too close to call. FWiW 24.23.197.43 (talk) 02:00, 24 February 2010 (UTC)[reply]
You forget an important difference. A smoker breathes in cigarette smoke about 10-20 times per cigarette (10-20 puffs). With about 10-20 cigarettes per day, that's about 300 breaths. The surrounding air, with all its exhaust fumes, you breath in constantly, every breath you take, day and night. With 20 breaths per minute that would be about 30.000 breaths per day. That's 100 times more! And quite often, constant exposure to something unhealthy is in the long run more damaging than more severe, but interrupted exposure. Of course, smokers often breathe in their own 'second hand smoke' as well, but that's a bit harder to assess (and becoming less common with smokers stepping outside for a smoke). DirkvdM (talk) 16:31, 24 February 2010 (UTC)[reply]
A few years ago this was a newsitem in the Netherlands. A research had shown that children who grew up (or went to school) near busy roads had lung problems which were sometimes very severe. It was said that their lungs were like those of a long time chain-smoker. I can't remember if that was just the worst cases, though. If I remember correctly, they spoke of a reduction in lung functionality of 10%.
However, that's the Netherlands, where most people live near the sea (Randstad). In the UK, you've got London, a metropolis that exceeds anything we have in the Netherlands in terms of size and congestion, and in the centre of which you are not likely to get much relief from a sea-breeze. Cities have always been an unhealthy surrounding. I just wonder if it isn't getting worse. DirkvdM (talk) 16:26, 24 February 2010 (UTC)[reply]

what is the most likely avenue for curing cancer?

What, in specific terms, is the most likely route being pursued for curing cancer? 84.153.231.223 (talk) 00:52, 24 February 2010 (UTC)[reply]

Every type of cancer is different, and there isn't going to be a single approach that works for all of them. The most promising approach is usually some combination of chemotherapy, radiation, and surgery. Once a cancer has metastasized, though, it seems to be generally incurable by any approach that is currently envisioned. Looie496 (talk) 01:34, 24 February 2010 (UTC)[reply]
Come on, Looie -- certainly the inquisitor was not asking for an explanation of current methods -- being pursued suggests a look into the future. So while we do not make predictions here at the ref desk, I would say the phasing out of all three methods mentioned by Looie would be quite welcome for pretty much the same reason: the target is too large. Chemotherapy works by targeting speedily growing cells, which certainly includes malignant cells, but also unfortunately includes those found in hair follicles and reproductive organs, among others. The effects of the medication are also embraced by unspeedily growing cells. Radiation therapy targets any and all cells, and the focus in this realm has been to narrow the beam and decrease the effective dose so that only the most targeted cells die -- but this again is not a perfect method. While surgery is great at removing the entire lesion assuming the lesion hasn't metastasized, clean margins are necessary and resections can often leave the cancer patient severely disfigured, although still alive. If I'd have to say what the pursuit of cancer research is in terms of defined therapy, I'd think it would be a more refined target derived from the specific type of cancer cell, so that the therapeutic agent would be a poison for only that group of cells but as harmless as sodium Na+ for the remainder of the body's cells. DRosenbach (Talk | Contribs) 01:51, 24 February 2010 (UTC)[reply]
You really think sodium metal is harmless for human cells? :-) 24.23.197.43 (talk) 02:06, 24 February 2010 (UTC)[reply]
You got me -- I was too lazy to find the <"sup"> <"/sup"> button :) DRosenbach (Talk | Contribs) 05:14, 25 February 2010 (UTC)[reply]
I took being pursued to mean being actively worked on, as opposed to being fantasized about. Regards, Looie496 (talk) 02:12, 24 February 2010 (UTC)[reply]
Only very recently has medical science discovered the connection between viruses and cervical cancer as well as at least one other type (I read about it recently, but can't remember which). The 20th Century bias toward looking for environmental, genetic, psychological and "life-style" causes of cancer led to an absence of reserach into viruses and bacteria as causes of cancer (or, for that matter, of all other chronic illnesses). (Remember when ulcers were caused by a bad attitude? Thanks, Sigmund. Meet my friend Dr. Pasteur.) It may be that the most fruitful preventative measure against cancer will eventually be innoculation, anti-biotics, and the like. See the book (circa 1999) Plague Time, by an Amherst College professor of evolutionary virology whose name escapes me. 63.17.65.39 (talk) 02:46, 24 February 2010 (UTC)[reply]
It's odd that people still talk about finding "a cure for cancer". We already have several cures for cancer. A very large proportion of people diagnosed with cancer these days recover fully (exactly what proportion depends on the type of cancer). As with any disease, we can expect our cures to improve over time or be replaced by entirely different cures (more effective, fewer and less serious side effects, cheaper, etc.). There is no way to predict what the improved or new cure would look like. --Tango (talk) 02:59, 24 February 2010 (UTC)[reply]
As Looie points out, the fact that pretty much every cancer is different means that different strategies are being pursued for different types of cancers, and each has different chances of success. A few examples:
  • for cancers that are highly associated with viral infection, vaccines hold great promise. The National Cancer Institute says, "Widespread vaccination has the potential to reduce cervical cancer deaths around the world by as much as two-thirds..." which is pretty promising (though is dependent on the usual vaccine caveats).
  • Another use of vaccines is against markers of the cancers themselves, see Stimuvax as a promising example of this.
  • The rapid decrease in cost, and development of third generation DNA sequencing, means we are approaching the time when it may be feasible to sequence the genome of individuals. Thus instead of being told "you have liver cancer", your doctor may be able to determine the exact mutation that has caused your liver cancer. This will make targeted therapy more effective and opens up the door to personalized medicine. It could also identify those at high risk of certain cancers, permitting earlier screening and detection, leading to higher survival rates.
  • Some companies have tried using viruses to kill cancer cells specifically, or use viruses to carry targeted chemotherapeutics. One of the early examples, Onyx-15, was not very successful but the concept seems to be working and other systems are in development.
There are many, many more (see Experimental cancer treatment for some examples), but comparing them is a bit like comparing apples and oranges and asking which is better for you. And if anyone knows for sure which is going to prove to be most successful, then they should invest in the company that is developing it and wait for the money to roll in. Rockpocket 03:31, 24 February 2010 (UTC)[reply]
"Plague Time" (referred to above) was written by Professor Paul W. Ewald. A reviewer from The Science Advisory Board writes: "Though there has been much investment in studies of genetic causes of disease, the most significant returns were generated on investments made in the studies of infectious diseases. Polio, measles, hepatitis, liver cancer, ulcers and cervical cancer were all controlled or shown to be controllable during the last half of the twentieth century through the control of infectious agents. This track record provides a sense of how to invest in our future: focus on the germs." http://www.scienceboard.net/resources/bookreviews.asp?cat=1&book=159 63.17.65.39 (talk) 05:08, 24 February 2010 (UTC)[reply]
I keep referreing to this comic: [1] for a quick explanation of the issue. It gets the point across EverGreg (talk) 09:22, 24 February 2010 (UTC)[reply]
It's a question of inputs versus outputs. Less noxious gases breathed in, less questionable food and drink, more fresh air and fresh food. That will be the avenue to cancer disappearing. In a lab? Oh please! Vranak (talk) 01:57, 25 February 2010 (UTC)[reply]
How would doing those things prevent infection by the viruses correlated with cervical cancer and liver cancer? (It took a long time to persuade the mainstream that stress doesn't cause ulcers ...) The "life-style" theories may be a post-Freudian, puritanical 20th century bias, now moving toward a dead end ... as viruses, bacteria, and other infectious agents are ignored until, as in cervical and liver cancer, they become too obvious to ignore. 63.17.57.56 (talk) 02:35, 25 February 2010 (UTC)[reply]
I will simply state that the human body is an extremely capable machine. It fights off degeneration for ages and ages and ages. It takes a lot to bring down a man -- about 80 years of sucking in nasty houses gases, most notably. Vranak (talk) 04:04, 25 February 2010 (UTC)[reply]
Whether healthy living is the "avenue to cancer disappearing" is very controversial. The reality is pretty much every environmental factor - even so-called "healthy ones" - can have mutagenic potential when you add up exposure over a lifetime. You may be able to combat some cancers by shutting yourself in a controlled, sterile room for most of your life away from mutagens and infectious agents, but good luck convincing anyone to do it. You will certainly be able to decrease your personal risk of some cancers by adopting a healthier lifestyle and avoiding risky behaviours, but that is hardly going to abolish all cancer. Moreover, with our increasing population and pollution levels, it simply isn't realistic to expect that human population can address cancer by returning our planet to an idyllic garden of Eden, spending out time in the fresh air eating organic food and avoiding pollutants. Ironically enough, if we did that rates of skin cancer would increase (unless we all used strong sun blocks which, incidentally, are developed in the lab). Like it or not, lab researched therapeutics, in co-ordination with education on healthier lifestyles, is the most realistic chance we have to combat cancers.
Finally, I take issue with your suggestion that our body "fights off degeneration for ages and ages and ages".
80 years is just a trifle to you then is it? Well, very well! I'm 30 and it feels like I've been around for an incredibly long time. Ten thousand days in fact. Maybe it's just that I have a clearer memory than most. Vranak (talk) 19:22, 28 February 2010 (UTC)[reply]
It doesn't. Degeneration (cellular, genetic, physiologic) is a continuous process that occurs pretty much throughout our lifetimes, certainly from our twenties onwards. Its called aging and no amount of healthy eating and fresh air will stop it. Cancer is the ultimate conclusion of that process. You pick up mutations in the very process of living your life, and eventually you will get one or two that hit a tumor suppressor gene or an oncogene. Unless something else gets you first, everyone on this earth will get a cancer eventually. It is completely inevitable, which is why it is so difficult to "cure" cancer. Rockpocket 20:59, 25 February 2010 (UTC)[reply]
Your pessimism and defeated spirit have tainted your thinking. The body has DNA repair mechanisms. Earl Grey tea (specifically, bergemot oil) stimulates those in fact. Vranak (talk) 19:25, 28 February 2010 (UTC)[reply]
See deoxycholic acid and Pacific yew. ~AH1(TCU) 00:59, 28 February 2010 (UTC)[reply]

bleach cover up

there is an employee of Clorox editing the bleach articles trying to minimize the risks of bleach and the carcinogenic and caustic nature of it. can someone stop him and revert his edits? —Preceding unsigned comment added by Thekiller35789 (talkcontribs) 03:29, 24 February 2010 (UTC)[reply]

Try asking here: Wikipedia:Conflict of interest/Noticeboard. --Tango (talk) 03:39, 24 February 2010 (UTC)[reply]
it would also help for you to give their user name so that we can check it out. --Ludwigs2 05:17, 24 February 2010 (UTC)[reply]

it is user http://en.wikipedia.org/wiki/User:GVB012009 —Preceding unsigned comment added by 67.246.254.35 (talk) 00:19, 27 February 2010 (UTC)[reply]

Does gasoline float on water? Will it dissolve sugar?

Will water, for example in a gas tank, float on top of the gasoline or sink to the bottom (and why)? Also, does sugar dissolve in gasoline, or will it stay granular? I think sugar will not dissolve due to the non-polar nature of sucrose and gasoline. —Preceding unsigned comment added by 161.165.196.84 (talk) 08:32, 24 February 2010 (UTC)[reply]

Gasoline floats on water, and sugar does not dissolve in gasoline. Read the Wikipedia article on solubility for more information on this. 217.158.236.14 (talk) —Preceding undated comment added 11:11, 24 February 2010 (UTC).[reply]
Snopes.com: Sugar in the Gas Tank might be of interest to you. AlmostReadytoFly (talk) 11:42, 24 February 2010 (UTC)[reply]
Snopes says "No", but The Straight Dope claims to have run the experiment and gotten a "Yes". Followup: are you sure sugar in the gas tank will wreck a car engine? APL (talk) 16:31, 24 February 2010 (UTC)[reply]
So we need Mythbusters to break the tie... --Tango (talk) 18:30, 24 February 2010 (UTC)[reply]
I note that the Straight Dope article notes that it clogs the fuel filter sock, exactly the result described by Snopes. They agree that sugar won't destroy an engine, and they agree that sugar can stop a car from operating. So I don't see a need for a tiebreaker, although giving the Mythbusters new excuses to destroy stuff is always entertaining. — Lomn 18:57, 24 February 2010 (UTC)[reply]
The Mythbusters did do "sugar in the gas tank" (I think it was episode 15) - and it didn't work. The engine ran just fine. They tried a bunch of other things too. What did finally kill the engine was bleach. SteveBaker (talk) 03:46, 25 February 2010 (UTC)[reply]

Electrolysis and Photolysis of Water

If you use electricity to split saltwater into hydrogen into oxygen, what would happen to the salt in the saltwater? Would it also split into sodium and chlorine or would it remain as salt?

If you use light or radiation to split saltwater into hydrogen and oxygen, what would happen to the salt in the saltwater? Would it also split into sodium and chlorine or would it remain as salt?

Bowei Huang 2 (talk) 10:07, 24 February 2010 (UTC)[reply]

When salt is dissolved in water, it already is split into sodium and chlorine ions. See the articles Ion and Solution. DVdm (talk) 10:59, 24 February 2010 (UTC)[reply]
WIth electrolysis you can get chlorine gas formed and sodium hydroxide at the other electrode. Hydrogen is formed in preference to sodium metal. Graeme Bartlett (talk) 11:21, 24 February 2010 (UTC)[reply]

If you use electricity, heat, light, or radiation, etc, to split saltwater into hydrogen and oxygen, then what happens to the sodium and chlorine in the saltwater?

Bowei Huang 2 (talk) 09:09, 25 February 2010 (UTC)[reply]

The above answers and electrolysis of water explain what happens when you use electricity (the sodium in particular will remain in the water, although you may get some chlorine and sodium hydroxide forming at the electrodes). Using heat or light or most forms of electromagnetic radiation will not split saltwater into hydrogen and oxygen (when you boil water you just get steam). (I'm not sure what high doses of ionizing radiation would do, but I wouldn't recommend you try it.) If you use the heat/light/radiation to generate electricity (in a steam turbine, photocell, nuclear power station, etc) then the answer for electricity applies. --Normansmithy (talk) 11:48, 25 February 2010 (UTC)[reply]

very hot water + some cold water = just warmed water?

If I want to heat a pot of water to a certain "quite warm" temperature, but overshoot my target and get a raging boil, will adding some cold water, and mixing, produce the equivalent "quite warm temperature" water as if I had just heated to that temperature directly? Or will the cold water "ruin" some aspect of the water? Please answer from both a scientific and practical standpoint. Thank you. 82.113.121.93 (talk) 12:41, 24 February 2010 (UTC)[reply]

Practically speaking, it can be a very bad idea to have hot and cold things together in glass or ceramic containers as thermal shock can cause them to break, but for a metal pot, addition of cool water to hot water will change nothing but the temperature and mass of the whole. Googlemeister (talk) 13:57, 24 February 2010 (UTC)[reply]
Adding to the practical warning, it's not so much an issue in this case. You're likely to encounter thermal shock issues if you plunge a raging hot container into an ice bath, or dump boiling water into a frozen glass, but adding cold water to a hot pot full of hot water will avoid the effect -- there's enough thermal inertia there to avoid the shock.
On the other side of practicality, some people claim that their boiled tap water tastes different from their unboiled tap water. They might consider such mixing as you suggest to be "ruining" the water, and from their subjective standpoint, that's fine. Note, however, that it's strictly a case-by-case basis. If mixing the water bothers you, don't do it. An alternative option would be to dunk a plastic bag full of ice water into the hot water; that will generate the cooling effect without mixing the water. — Lomn 14:02, 24 February 2010 (UTC)[reply]
I have no idea what you mean by 'ruining' the water (do you mean for making tea or something?).
But the temperature-thing is easy. If you add an equal amount of water then the resulting temperature will be the average of the two. For example, if you have boiling water that'll have a temparature of 100 C (a 'raging boil' just means that it is working harder at losing the heat added to it by the heat-source). Suyppose it's 1 l. If you add 1 l of water at room temperature, say 20 C, then the resulting temperature will be (100 + 20) / 2 = 60 C. If you add just 50 cl, then you get three parts, two of which are the boiling water, so it will be (2*100 + 20) / 3 = 220/3 = 73,8 C. If you add just 10 cl, it'll be (10*100 + 20) / 11 = 1020/11= 92,7 C. In other words, if you want just 'quite warm' water, say about 40 C, then you'd have to add more water at room temperature than the water in the pot.
But it would be easier to use ice-water (water in which some ice cubes have been floating for a while without completely melting). This is more practical because you can create both temparatures without a thermometer (one with a boil the other with ice). And it makes for easier mathematics as well because you've got a 'point zero' and the other number is 100 (note that this only works for Celsius, not for Fahrenheit). To get at a certain temperature, divide 100 by it and you get the proportion of boiling water you need in the mix. For example, to get 25 C, that has to be 25/100 = 1/4. So the proportion of ice water has to be 1 - 1/4 = 3/4. So 1 part boiling water, 3 parts ice water. If you want 40 C it's 40/100 = 4/10 and 1 - (4/10) = 6/10, so 6 parts boiling water and 4 parts ice water.
I'm sure there's an easier way to explain this, but I hope this helps. DirkvdM (talk) 15:50, 24 February 2010 (UTC)[reply]
Just a practical point: use only the water that had the ice in it. Don't add the ice; it counts as "below 0 °C" even if its temperature is exactly that because of the heat of fusion. --Tardis (talk) 16:41, 24 February 2010 (UTC)[reply]
Our water memory article is good reading. Comet Tuttle (talk) 17:51, 24 February 2010 (UTC)[reply]
For a sufficiently warped definition of "good"! --Tango (talk) 22:09, 24 February 2010 (UTC)[reply]
Avid tea drinkers claim that over-boiling the water drives out oxygen and perhaps other dissolved gasses and makes it taste 'flat'. There may be some truth to this...but shaking it up with some air ought to solve that pretty quickly so "ruined" is perhaps a bad choice. SteveBaker (talk) 03:23, 25 February 2010 (UTC)[reply]

Surface of Gas Giants

While reading 2010: Odyssey Two, I remember ex-David Bowman observing that Jupiter's core was a diamond the size of Earth, due to the intense pressure on the gases in that region. I, personally, don't believe there is enough carbon in the Jovian atmosphere to create a diamond, but is it at all possible that this theory could be true? MMS2013 14:19, 24 February 2010 (UTC)[reply]

Unlikely. ~ Amory (utc) 15:32, 24 February 2010 (UTC)[reply]
But there are serious "diamonds in space" hypotheses: Neptune#Internal_structure, BPM 37093. -- Coneslayer (talk) 15:42, 24 February 2010 (UTC)[reply]

Aspirin from a tree?

I recently went on a jungle trip, on which my guide showed me a tree the sap of which was supposedly good for stomach-aches. He let me try a bit and it tasted very much like aspirin. At the aspirin talk page someone mentioned that "salicylic acid is similar to the acetylsalicylic acid in aspirin, and is found naturally in some plants, such as the willow tree." So is that all basically the same stuff? Or might a new source of a similar chemical still be useful for medicine? (In other words, can this make me and/or the locals very rich? :) ) DirkvdM (talk) 15:09, 24 February 2010 (UTC)[reply]

See salicylic acid. It is a well-known product, not a new discovery. Of course, you could get rich off it. There are a lot of people who sell plants and herbs that don't do much and make good money. All you need to do is find someone to buy it.
I don't know what plant you were shown, but the substance in willow bark essentially is asprin. Acetylsalicyclic acid is just a synthetic version - it is much cheaper to use fancy chemistry to make it than to grow lots of willow trees (and I think it is slightly more effective/has fewer side effects). Using willow to treat pain was well know long before modern chemistry. --Tango (talk) 16:44, 24 February 2010 (UTC)[reply]
Actually, no — that's what I used to think too. In fact it's what pretty much everyone used to think. But they set me straight on the talk:aspirin page.
Aspirin has a different mode of action from salicylic acid (the stuff in the willow trees). Aspirin irreversibly acetylates some enzyme, probably COX but I'm not sure about that. Obviously salicylic acid can't acetylate anything, as it has no acetyl group. --Trovatore (talk) 21:46, 24 February 2010 (UTC)[reply]
It's also not true that aspirin is easy to make than salicylic acid. Aspirin is made by acetylating salicylic acid. Aspirin is was preferred mostly because it attacks the stomach much less strongly. Algebraist 21:56, 24 February 2010 (UTC)[reply]
Well, no, it's preferred because it acetylates COX, whereas salicylic acid doesn't. Salicylic acid has some independent effect; I don't know what it is. I don't know whether it's an anti-inflammatory at all. --Trovatore (talk) 21:58, 24 February 2010 (UTC)[reply]
Sorry, I should've made it clear I meant that's the reason aspirin was preferred when it was first devised. I think you're right that salicylic also lacks the non-analgesic effects that were discovered for aspirin later. Algebraist 22:03, 24 February 2010 (UTC)[reply]
The "new source of a similar chemical" would have to be more effective, or have fewer side-effects (or preferably both) if you want to make your fortune out of it (without "quackery"). If you have found a new "wonder-drug", then you must take out world-wide patents as soon as possible. You should be aware, however, that most similar chemicals have already been tried and rejected long ago. Dbfirs 21:40, 24 February 2010 (UTC)[reply]
Ah, 'similar drugs' is what I was getting at. So there are variations on the drug. (Also see the polymorphism section, although I suspect that that gives just a limited range of variation.) Might there be a list somewhere of plants and trees that contain similar drugs?
I suppose one first question is whether this is a willow tree. Comparing my pictures with these, it may very well be, although I'm by no means an expert. Also, it had shoots that look rather willowish to me. I assumed willows didn't grow in the tropics, but they seem to grow just about anywhere (just mostly in moderate climates). But even if it is a willow tree, there appear to be many species of willow, with lots of variation (not surprising since they grow just about anywhere) and this might be an uncommon one (possibly even unknown to western science, given where I found it). Do they all have the same salicylic acid? If so, and if this is a willow tree, then I suppose that's the end of it.
One more thing, though. Which drugs taste like aspirin? Is it the salicylic acid that gives it the typical taste or the acetyl, or what? DirkvdM (talk) 09:25, 25 February 2010 (UTC)[reply]
You should bear in mind that it costs a pharmaceutical company like Bayer (very roughly) about $500,000,000 (yes, that's five hundred million US dollars) to take a "candidate molecule" all the way from discovery of its existence (or its invention), through all the various testing and trials and licencing processes required by governments, to being a marketed product. The subsequent profits it makes for the company also have to cover the costs of all the (many more) candidates that didn't make it all the way, a roughly comparable sum.
Because pharmaceutical companies are working with such vast sums they can afford to employ extensive research staffs who are well aware of the potential for new drugs in locally known remedies such as this; it's therefore very probable that this one has already been, or at best is being, investigated by one or more of them [Disclosure: I used to work at #12 on (both of) the above-linked lists].
On the very slim chance that you have stumbled upon something not yet investigated, you might consult (probably at considerable expense) a lawyer specialising in such matters about how to retain some interest in it, but consider also that most people and legal systems would probably consider that the indigenous people from whom you learned about it would have a far greater claim than you on the fruits of its exploitation.
There are of course less currently regulated and consequently less costly (but also less lucrative) areas of "hebal", "folk" and other "Alternative medicines." Again, it would be surprising, if less so, if this particular local remedy were not already known to such industries. It seems likely from recent trends and events [2] that regulation of these (in the "Western World") is going to be tightened up in future. 87.81.230.195 (talk) 16:23, 25 February 2010 (UTC)[reply]
Some studies link Asprin to Reye's syndrome. Also, there are plenty of "herbal" remedies such as those for Ovarian and breast cancer from the Pacific yew and for leukemia from Madagascar Periwinkle seeds. ~AH1(TCU) 00:55, 28 February 2010 (UTC)[reply]

r-tree structure for image retrieval

Hi

please, I have 16-dimension color histogram , i need to use r-tree indexing in order to speed up the image retrieval. how can I determine the MBR in the tree?, and How the overlapping happen? ,how many level I need to represent r-tree? please any help زينب —Preceding unsigned comment added by Dania2009 (talkcontribs) 15:15, 24 February 2010 (UTC)[reply]

Have you read the article R-tree? Cuddlyable3 (talk) 23:12, 24 February 2010 (UTC)[reply]

rebok

is castro wearing rebok sneakers in this pic?

http://en.wikipedia.org/wiki/File:Fidel_Castro8.jpeg  —Preceding unsigned comment added by Thekiller35789 (talkcontribs) 15:22, 24 February 2010 (UTC)[reply] 
They are obviously trainers of some kind - but the photo was taken in 2003 and because Reebok change their designs at least several times a year - it would be very hard to find an exact match for them by searching for images on the web to get a match for the design. However, he clearly likes to wear sneakers here, for example. He's also not above advertising some western brand names either: Adidas!. So I doubt we can say that he was wearing Reeboks - but it's very likely some kind of brand-name sneaker. SteveBaker (talk) 00:57, 25 February 2010 (UTC)[reply]

why is the universe so dumb?

It's obviously a huge oversight that just be speeding around for a while relative to the other, two twins can end up with arbitrarily different ages. That obviously doesn't make much sense. Therefore, the Universe is very stupid. My question is: why is it so stupid? Why isn't it more sensible? 82.113.106.92 (talk) 16:32, 24 February 2010 (UTC)[reply]

Your argument is just an opinion, not a fact. What you are asking is "Why do I think the universe is stupid?" From your statement, the best answer that I can give is "Because you don't understand it."
Time dilation is an inevitable consequence of the speed of light being the same to all observers. There isn't really a reason for it, though, the universe is just the way it is. --Tango (talk) 16:45, 24 February 2010 (UTC)[reply]
well isn't it pretty dumb that if you are a photon of light going the opposite way of another photon of light, and you're both travelling at c, then you are not moving away from the other one any faster than if the other one were a brick wall? That seems really dumb to me, but if you say that it is only my opinion, why don't you give me ONE good reason why it's not dumb? I can't see any reason why it's not dumb of the universe to be such that hurling away at c from a brick wall doesn't move you away from it any faster than hurling away at an object going 186,000 miles per second in the opposite direction. I'm open to any reason why this shouldn't be a really dumb set of laws... (I don't mean that they don't apply, but that it's dumb that they should, that our universe should be so shoddily hacked together in this way). Thanks. 82.113.106.92 (talk) 17:09, 24 February 2010 (UTC)[reply]
Stupidity is a factor of intelligence, which means you're suggesting the universe is either a life form, or was created by one. Science can't answer those kind of questions. Vimescarrot (talk) 17:18, 24 February 2010 (UTC)[reply]
The universe is wise, which has no bearing on intelligence. --Ludwigs2 17:20, 24 February 2010 (UTC)[reply]
Is time dilation really that much less counter-intuitive than, say, the doppler effect or redshift? In other news, where do you want us to go with your assertion that you find the universe dumb, beyond "so what; get a clue"? --Tagishsimon (talk) 17:25, 24 February 2010 (UTC)[reply]
The universe is not dumb, and neither are the laws of relativity (or, at the other end, the ones of quantum dynamics). It's quite literally us, who only experience "the middle world" who form wrong assumptions by looking at a small part of the world with bad focus and then expect our assumptions to apply to all of reality. Newtonian physics is not the law and relativity the exception, rather, relativity is the law and Newton's laws of motion are an approximation that works in some limited case. --Stephan Schulz (talk) 17:28, 24 February 2010 (UTC)[reply]

(ec) Come on, obviously I'm not talking about 'intelligent' design in the Universe. I mean, in human terms, the way things are are pretty dumb. Actually 186,000 miles per second isn't that fast. It takes an easily discernable amount of time just to ping someone in China and get their response - it is physically palpable lag. And that's on just a local, Earthly basis. On a galactic basis, this universal speed limit is ridiculous. I'm not saying "SOMEBODY" made the speed limit be 186,000 miles per second. I'm saying, it's a stupid state of affairs, regardless of whether anything intelligent (or lack thereof) caused it. It's just stupid. But, for anyone who actually is taking the effort to understand what I'm SAYING here, I'm open to anything you know that makes the above aspects of the universe I mentioned actually be not-stupid, but pretty cool. Here is an alternative way of phrasing it: If reality really were that conceived by classical physics people thought we were living in in the 18th century, NO ONE, but NO ONE, would wish "sigh. If only there were a universal speed limit of 186,000 miles per second. If only you could never move, even in the vacuum of space, even close to that speed. If only twins could end up with different ages if one takes a fast trip." Why would anybody want that? Nobody would want that: it's dumb. Unless you know of some reason why somebody WOULD pine after the state of our universe, if they were like us but lived in a nonrelativistic place. Also, relativism is just one aspect of our dumb universe, don't even get me started one quantum mechanics. One thing at a time: is there anything good or nice about living under relativistic physics? Or, is it merely dumb for the reasons I outlined? Thanks. 82.113.106.92 (talk) 17:39, 24 February 2010 (UTC)[reply]

This thread should probably be collapsed for trolling, but I have a good answer. I like living in this awesome relativistic universe because if the speed of light constant and time dilation were any different, it would affect things like the formation of stars and galaxies in unpredictable ways, and human life would most likely not exist. Comet Tuttle (talk) 17:48, 24 February 2010 (UTC)[reply]
That's a very good answer, actually, despite the fact that you dismiss my question as a troll. I hadn't thought of that answer. However, that is not due to real stupidity on my part, but because of a lack of science education. All of the humanities (human affairs) are inconsequential spots in the formation of the stars, and so the fact that we are inconvenienced by ping times, for examples to a planet as close as Mars, is nothing compared with the benefit of existing. I guess it's like some really stupid human laws, that nevertheless serve an important purpose, and "but for" them, the place would go to hell in a handbasket. 82.113.106.92 (talk) 17:54, 24 February 2010 (UTC)[reply]


It seems that the OP is equating "counterintuitive" with "dumb." This is a huge fallacy. First of all, things are only ever intuitive because you have daily experiences with them, and can perceive events and judge generally what you expect should happen. But physics is the quantitative analyis of the mechanisms of the universe at many different scales. Relativity in particular is concerned with the physics of very large masses, very high energy, very fast speeds (and similar). Since humans are small compared to the scales necessary to observe many relativistic effects, we have no daily experience with them; nor have we evolved biological sensors which can make meaningful sense of those relativistic effects. The same applies to quantum mechanics - which deals with very small energy scales, length scales, and time scales. Again, the behaviour of the universe is very different at such scales - so it seems to contradict your "gut sense" of what should happen. But if you spend a lot of time observing things at quantum or relativistic scales (which means learning to use the mathematical machinery and understanding the experimental observations that are available) - then you will see that the universe actually is quite ordered and follows well-defined rules. Nimur (talk) 18:50, 24 February 2010 (UTC)[reply]
It sure seems to follow rules given the current level of sensitivity of equipment. Is it possible that finer analysis may reveal that it does not, in fact, follow any rules at all? 82.113.106.89 (talk) 19:23, 24 February 2010 (UTC)[reply]

Also: I don't equate "dumb" with "counterintuitive" -- I call it dumb because no one would ask for them in their absence, but there are a lot of counterintuitive things that people DO ask for, in the absence of any such effect. Two great examples are replication and teleportation, neither of which make any sense on an intuitive level, yet both of which are commonly requested. In other words, every single one of the aspects of the universe"I've described is more readily described by its users as bugs, not features; meanwhile, actually requested features are totally absent. That's what I'm getting at: that what we get, is a wet blanket. I'm especially talking about relativistic effects. They are not useful, but on the contrary, dampen many ambitious plans (such as the trillion mile per second rocket etc). 82.113.106.89 (talk) 19:27, 24 February 2010 (UTC)[reply]

"My own suspicion is that the universe is not only queerer than we suppose, but queerer than we can suppose." J.B.S. Haldane. --TammyMoet (talk) 18:57, 24 February 2010 (UTC)[reply]
Why would you expect the universe to be how human science-fiction writers want it to be? --Tango (talk) 19:43, 24 February 2010 (UTC)[reply]

As you can see we already have a pretty long thread and we haven't even started answering your question yet mainly because of your poor choice of words. I'm sure there is a lesson somewhere in there... Dauto (talk) 20:32, 24 February 2010 (UTC)[reply]

I know you were exaggerating, but actually you have started answering the question, viz. Commet Tuttle's "if the speed of light constant and time dilation were any different, it would affect things like the formation of stars and galaxies in unpredictable ways, and human life would most likely not exist". That's the anthropic principle, and kind of a good answer. By the way, you said I used a poor choice of words, but I don't really have better ones for the question and feeling that I'm addressing. What would you suggest I say, given the clarification to the question I've since given? 84.153.235.131 (talk) 21:20, 24 February 2010 (UTC)[reply]
Words like 'frustrating', 'annoying', or even 'unfair' would have worked better than stupid and dumb. Dauto (talk) 22:49, 24 February 2010 (UTC)[reply]
A complaint that something is "obviously a huge oversight" should be addressed to the one you think is responsible. Unfortunately there Ain't Nobody Here But Us Chickens. Cuddlyable3 (talk) 23:09, 24 February 2010 (UTC)[reply]
Personification is not science. If you wanted the science, ask science questions. If you wanted to know why the Universe is stupid to you...that would be psychology, one of the Humanities? The question, as you asked it, seemed to be applying psychology (concepts of intelligence, or lack thereof) to physics. The two are incompatible. Vimescarrot (talk) 23:27, 24 February 2010 (UTC)[reply]
I once read a short sci-fi story (Far Centaurus by A. E. van Vogt, I think) where the means of travelling faster than light involved work in the field of "electron psychology". What this meant was never really explained... --Tango (talk) 01:02, 25 February 2010 (UTC)[reply]
The problem with the universe is not with it - but with us. We've evolved to clamber around in trees, chase after small furry animals, clonk them on the head so we can eat them and frolic through the grasslands picking delicious nuts and berries. Nothing we've evolved to deal with goes faster than maybe 100mph and no event that we ever care about takes less than the literal blink of an eye. Nothing weighs more than a smallish planet - nothing is (perceptably) larger than however far you can run in a day. So when we find that the rules at scales that our brains have evolved to cope with do not apply for anything that's very heavy, very large, very fast, very tiny or very brief, we're not really equipped to handle that. When we're exposed (through our own science) to things that are as downright weird as time dilation or quantum theory - we're inclined to the kinds of response that you're feeling. It's not at all a strange or inexplicable reaction to how the universe is...it's simply that (for most of us) the boundaries of our intellectual knowledge has outpaced our ability to truly internalize those things. The universe is actually rather cool (I wouldn't say "dumb" or "intelligent") - some of this weird stuff is really useful. Without all of that quantum theory, most of the fancy electronics that we love so much wouldn't be possible. The speed of light thing is a bit of a bummer for the science fiction fans who really wanted to go and explore other star systems - but you've gotta admit it's interesting. SteveBaker (talk) 00:41, 25 February 2010 (UTC)[reply]
The real question is, why do you care if the Universe is stupid? If you are so smart then it shouldn't trouble you in the slightest. Only if you were also stupid would other people's stupidity (or universal stupidity) be a hassle. Vranak (talk) 01:56, 25 February 2010 (UTC)[reply]

The OP writes "don't even get me started on quantum mechanics." Without quantum mechanics, the protons in the sun wouldn't fuse because they would invariably repel each other (both being positively charged; that's intuitive and sensible, right? -- for an "intuitive" but wrong analogy, think of trying to force "the wrong ends" of two magnets together). The hydrogen in the sun wouldn't produce helium, and so on. It would be very cold and very dark and none of us would exist. Fortunately, quantum tunneling and and velocity distribution allow hydrogen protons to fuse (after a few tens of millions of years of hanging out in the sun). Thanks, stupid quantum mechanics! 63.17.57.56 (talk) 02:56, 25 February 2010 (UTC)[reply]


Theology is adept at discussing vaguely phrased and confusing questions. In your question we see a parallell with the well-known Problem of evil (POE) in your Problem of stupidity (POS). Some of the answears to the POE can be easily amended to answear the POS, such as mankinds limited ability to assess the universe or the existence of free will. Or maybe the POS is a variant of the Omnipotence paradox? You could start with that and move on to the Euthyphro dilemma with some very interesting conclusions. (There must be a forum for such ideas somewhere :-) EverGreg (talk) 09:27, 25 February 2010 (UTC)[reply]
Adept at discussing - certainly. Able to come to a true, useful or even comprehensible answer? Never. SteveBaker (talk) 04:06, 26 February 2010 (UTC)[reply]

I think that the best answer so far is form Comet Tuttle. All of the physical constants of the universe (not just the speed of light) are important for determining the state of the universe. If any of them were different we probably wouldn't be here. If I fell out of a tree and broke my leg, I might say gravity is 'dumb', but I don't, because without a specific gravitation constant there may very well be no tree, no earth, no me, and quite possibly no universe. 24.150.18.30 (talk) 03:26, 27 February 2010 (UTC)[reply]

Salt ionizing in water

If salt is ionized in water, why does the water still taste salty? I know that sounds dumb, but if the salt gets separated into Na+ and Cl-, wouldn't it stop being salt and stop having its properties? Maybe not all of the salt gets ionized? Or maybe I just don't understand the process fully. —Preceding unsigned comment added by 160.36.39.222 (talk) 16:50, 24 February 2010 (UTC)[reply]

Don't forget that any undissolved salt entering your mouth will dissolve soon enough in your saliva. So judging from that alone, it seems to me that all you ever taste is the ions. Vimescarrot (talk) 17:21, 24 February 2010 (UTC)[reply]
(edit conflict) It is the sodium ions that cause the salty taste - our article on taste says "Saltiness is a taste produced primarily by the presence of sodium ions". Other alkali metal ions produce a similar taste. I doubt it is possible to know what undissolved salt tastes like - before it gets anywhere near your taste buds, it will have dissolved in your saliva. Gandalf61 (talk) 17:22, 24 February 2010 (UTC)[reply]
Other alkali metals tasting roughly the same is why potassium is often used to replace the sodium in low-sodium salt. (Not that potassium is any better for you in large quantities than sodium, unless you have some medical condition that makes sodium really bad.) --Tango (talk) 18:08, 24 February 2010 (UTC)[reply]
I'd say very roughly. I can easily distinguish KCl from NaCl. KCl is subjectively much more "sour".
It's too simple to compare K to Na as "which one is better for you". The main issue is that most people eat far too much sodium. On the other hand it takes (IIUC) significantly less potassium to outright kill you (quickly, as opposed to via hypertension, which generally takes years or decades to be lethal). --Trovatore (talk) 23:39, 24 February 2010 (UTC)[reply]

Galactic Plane and Rotation

Relative to the equator (or ecliptic) what is the angle of the galactic plane? Assuming the north pole of the earth faces upwards does the galaxy rotate CW or CCW? Is there some rule determining which way a galaxy rotates? TheFutureAwaits (talk) 17:23, 24 February 2010 (UTC)[reply]

See Galactic coordinate system, I think it answers all your questions except the last. The way a galaxy rotates will depend on the random motion of the matter than collapsed to form it. That motion will, due to random chance, have had a slight net rotation when you average it out. As the proto-galaxy collapses, conservation of angular momentum means the rotation gets faster and the galaxy flattens out into a disk. Which direction it rotates will be the same direction as that tiny net rotation the original cloud had, which is random. The same principle is what results in all the rotations in the solar system (rotations of planets on their axes, orbits of planets around the sun, orbits of moons around planets, etc.) which are almost all in the same direction (anticlockwise when viewed from above the north pole, if memory serves). The exact mechanism for the formation of galaxies is still up for debate, but what I've described should be accurate as far as it goes. --Tango (talk) 18:06, 24 February 2010 (UTC)[reply]

Numbers and diagrams only get you so far in getting a real gut feel for this. So how about thinking about it like this: The photo at right of the Milky Way was taken at a Telescope facility in Paranal in Northern Chile (More info about the photo comes from here). It was taken in the middle of winter and Paranal is well south of the equator - so we're somewhere "underneath" the planet. The bright dot in the middle of the photo is Jupiter. The laser beam is pointing towards the center of the galaxy. It takes a bit of mental gymnastics to get your head around that. But since Earth and Jupiter both lie in the plane of the solar system's ecliptic - and the photo was taken in the middle of the night - so the sun is somewhere off the bottom of the photo beneath our feet - so if you imagine a triangle with one point where Jupiter is, one where we're standing and another point way below our feet - then you can get a feel for how the plane of the solar system ecliptic doesn't line up well with the plane of the galactic disk. Hopefully, this conveys the fact that the solar system is tilted at about 85 degrees to the plane of the galaxy. When you look at the Milky Way, it fills about 100 degrees of your field of view - if you put your arms out to try to grab it(!) they make an angle bigger than a right angle. Imagine you're holding a gigantic round disk in your hands while they are 100 degrees apart - and that should give you a feeling of just how far on the outer edge of the galaxy we really are. SteveBaker (talk) 00:15, 25 February 2010 (UTC)[reply]

Where does that 100 degrees come from? The Milky Way surrounds us on all sides as a belt. That's 360 degrees. How much of it you can see at any given time will depend on where on the planet you are. The bit towards the centre of the galaxy is much more dramatic, of course. --Tango (talk) 00:35, 25 February 2010 (UTC)[reply]
Yeah - of course you're right - the galaxy is all around us. The number I quoted is for the bright stuff - and it's very approximate - no science behind it. If you get someplace where you can see the Milky Way clearly - it just looks like you could grab it with outstretched arms at a bit more than 90 degrees. The number "100 degrees" is mentioned at the link I pointed to here. SteveBaker (talk) 01:23, 25 February 2010 (UTC)[reply]

Thanks guys, this explains a lot regarding the angle. The only part I don't get is why the gravity of the galaxy wouldn't be enough to cause stellar systems to align their planets with the galactic plane. If the gravity is strong enough to make everything rotate around the galactic core then why aren't stellar plane's aligned? TheFutureAwaits (talk) 14:15, 25 February 2010 (UTC)[reply]

Looking at redshift for nearby galaxies would likely determine their rotation. As for the orbits of stars and their planes around galaxies, see Sun's orbit. ~AH1(TCU) 00:47, 28 February 2010 (UTC)[reply]

Weather and Axial Tilt

What would the weather and seasons be like on Earth of the axial tilt was 90 degrees instead of 23? Would the change have prevented life from evolving? TheFutureAwaits (talk) 17:26, 24 February 2010 (UTC)[reply]

The more I think about it there could be two answers couldn't there? If the "south pole" of a 90 degree axial tilt was pointing along the orbit of the Earth (aka 90 degrees from the direction of the sun) then there would be no seasons, just a 12 hour day night cycle right? If the "south pole" was facing the sun then there would be extreme seasons resulting in 6 months of darkness for every location on the planet. I imagine that would be enough to cause catastrophic weather likely preventing any life. Is my understanding correct? TheFutureAwaits (talk) 17:41, 24 February 2010 (UTC)[reply]

Not quite. The axis stays put as the Earth orbits the Sun so, relative to the Sun, it rotates once a year. This is what causes the seasons. Your two scenarios are actually just different parts of the same scenario. At one point the north pole will be pointing at the sun, then 3 months later the axis will be pointing along the Earth's orbit, then 3 months later the south pole would point at the sun, then 3 months after that it is pointing along the orbit, just in the other direction, and then 3 months later the north pole is back facing the sun. During spring and autumn you would get normal 12 hour days, during summer you would get constant sunlight and during winter constant darkness. It's just like what happens inside the Arctic/Antarctic circles in real life, but the circles will now be the equator so the whole northern hemisphere is the Arctic and the whole southern hemisphere is the Antarctic. --Tango (talk) 17:51, 24 February 2010 (UTC)[reply]
Somewhere above about 70 degrees of tilt it is predicted that the poles would have an annual average temperature higher than the equator (due to many months of 24 hour sun at the poles) and the equator would start to form an ice belt due to the generally oblique sunlight it sees most of the year. Dragons flight (talk) 19:04, 24 February 2010 (UTC)[reply]
Average temperature is irrelevant to life. Far more important is the extremes. The equator may be colder on average, but it would be much more stable, which is what is important. --Tango (talk) 19:45, 24 February 2010 (UTC)[reply]
That's not true. Average temperature matters a lot when you put snow and ice in the mix. Whereas many forms of life, e.g. deciduous plants and hibernating animals, have adaptations to specifically deal with seasonal variability. If the equator had permanent ice sheets (which is likely at a 90 degree tilt), then I think it would be rather barren of life the same way our polar deserts are relatively barren now. Stability matters, but so does staying away from 0 C for us water based lifeforms. Dragons flight (talk) 22:19, 24 February 2010 (UTC)[reply]
If you have an ice belt, I would expect the life to be underneath it where there would be liquid water. If the equator is always frozen there will be belts on either side that are seasonal and life could migrate between. --Tango (talk) 00:02, 25 February 2010 (UTC)[reply]
As for the consequences for life, it is difficult to say since we only have a single example of a planet where life can evolve. You certainly couldn't have life like we have, there would need to be major changes. The equator would be fairly stable temperature-wise, so I suppose life could evolve there. Some very simple life might be able to survive the massive temperature changes elsewhere or complex life could migrate (as many species do now). Exactly how great the temperature variations would be would depend on whether winds can transfer heat from the day side to the night side effectively. The issues are very similar to those found with tidally locked planets, like many of the planets we've found around other stars are thought to be. See Hot Jupiter for some information. --Tango (talk) 17:56, 24 February 2010 (UTC)[reply]
As I understand it, the current best theories for abiogenesis involve white smokers underwater. For this, plate tectonics (and hence a large hydrosphere) are more important than climate. What happens after that is a lot harder to predict, but I would trust evolution to find ways of coping with the different scenario. --Stephan Schulz (talk) 19:03, 24 February 2010 (UTC)[reply]
I assume that life first formed in the oceans, in which case as long as you don't freeze the oceans solid, I'd say the existence of life is still quite likely. Life on the surface would have to deal with greater extremes, but the oceans are a pretty good buffer for many forms of life. Dragons flight (talk) 19:04, 24 February 2010 (UTC)[reply]
No matter how the planet spins or points, there have to be some places that get a bearable amount of sunlight for a bearable amount of time. When the south pole points towards the sun, it cooks and the north pole freezes - but around the equator, there is a nice belt that gets a reasonable amount of sunlight. When the north and south poles line up along the path of the orbit, then at the equator, you have a fairly normal day/night cycle - so living on the equator would be kinda like living at the poles for us with months of perpetual twighlight followed by months of more normal conditions. The third extreme is when the poles point at right angles to the plane of the orbit...which is more or less what we have. Even if the planet isn't spinning at all, there are still places that get a reasonable amount of sunlight. Probably the worst case is not spinning but not tidally locked so that the nominal poles of the earth point toward the sun for months and then along the orbit for months - but even then, there are two points on the equator that get reasonable sunlight all year round. There ought to be liquid water no matter what (although not necessarily everywhere). So long as life has a way to get started, it's hard to imagine that between migration, hibernation and simply living in the right places, it could not evolve and continue. The most significant issue is what these peculiar arrangements do to weather and water - if it's enough to cause crazy energetic stuff to happen - then life might have a hard time of it. SteveBaker (talk) 23:51, 24 February 2010 (UTC)[reply]
I could imagine very strong currents in either the atmosphere, the oceans (under the ice) or both. Whenever you have localised heating of a fluid, you get convection currents. They could be really strong. --Tango (talk) 00:02, 25 February 2010 (UTC)[reply]

This is a really interesting subject. Steve, I'm having a hard time visualizing the two points on the equator you mentioned:"Probably the worst case is not spinning but not tidally locked so that the nominal poles of the earth point toward the sun for months and then along the orbit for months - but even then, there are two points on the equator that get reasonable sunlight all year round." Is there any chart or map which would explain this more clearly? In fact any graphic of any of these scenarios would be most appreciated. TheFutureAwaits (talk) 11:11, 25 February 2010 (UTC)[reply]

OK - so for some months of the year, (let's say around January) the planet's North pole faces the sun. Any point on the equator sees the sun on the horizon for months at a time whether the planet is spinning or not. The same for the months when the South pole is facing the sun (July). This is the same whether the planet is spinning or not. But in the intervening months, as the north-south axis becomes parallel to the direction that the planet is moving around it's orbit (April and October), then (if it's not spinning on its axis) there will be a circle of points running around the planet from one pole to the other via the equator that see the sun on the horizon. So during the course of three months - we go from one pole cooking (December) and the other freezing with the equator being in partial sunlight and therefore potentially livable - to a situation in April where (let's say) the Eastern hemisphere is cooking and the Western hemisphere is freezing with a circle of points going through both poles that is in partial sunlight. OK so now, imagine - as the planet moves between those two extremes, that circle of "reasonable temperature" gradually rotates from one of these positions to the other. However, there are two points on the equator where the April circle of reasonableness crosses the equator that are in partial sunlight all year round Everywhere else gets cooked and frozen alternately.
But if the planet is spinning about its axis - then in December and June, half the planet is frozen and the other half cooks - but in April and October, everywhere sees a 24 hour day/night cycle. In that scenario, anywhere on the equator gets normal days for six months out of the year - and perpetual twilight for the other six.
SteveBaker (talk) 04:00, 26 February 2010 (UTC)[reply]

Joseph Jenkins

is ther a link between jenkum and Joseph Jenkins? (humanure) —Preceding unsigned comment added by Thekiller35789 (talkcontribs) 17:57, 24 February 2010 (UTC)[reply]

This isn't really a science question. I'm trying to give you the benefit of the doubt, but it seems that your post is an effort to troll the desk so that people will search for shock sites. Little do you realize that the Ref Desk regulars are savvy internet-jockeys who will not be taken in by such antics. If you disagree with this point of view, and have a valid reason why you posted this to the Science Desk, please elaborate. Nimur (talk) 18:54, 24 February 2010 (UTC)[reply]


put my question back up. i was not trolling, your paranoid. humanure is something i do. its been around since 1990 and jenkum is a new thing. i wanna now if theres a correlation. —Preceding unsigned comment added by 67.246.254.35 (talk) 13:01, 25 February 2010 (UTC)[reply]

I have restored your question. Abecedare (talk) 16:45, 26 February 2010 (UTC) [reply]
Almost certainly "no". The word "Jenkem" seems to have originated in Zambia. Whereas the Joseph Jenkins of humanure fame is from USA.
It would be a really bizarre twist if Jenkem turned out to be not just urban legend, but complete hoax designed to parody Joseph Jenkins. A bit too obscure of a gag, though.APL (talk) 15:54, 26 February 2010 (UTC) copied reply originally posted on talk page. Abecedare (talk) 16:45, 26 February 2010 (UTC) [reply]
Urban dictionary[3] notes that jenkem is known on the streets as Sir Author Leroy Jenkems.Cuddlyable3 (talk) 16:55, 26 February 2010 (UTC)[reply]
To elaborate on APL's reply: while the reports of Jenkem abuse in US are almost surely an urban legend, its use in Lusaka, Zambia seems to have corroboration. See for example this WHO report (page 58 of the pdf), which says "Staff discovered that some were using 'jenkem' made by fermenting raw excrement, waste and/or sewerage over a few days and then inhaling it", where the staff refers to members of the Zambia Red Cross. The report dates back to 1996, more than a decade before the 2007 media interest in the US. So it's likely that the term and concept arose originally on the streets of Zambia, and that makes it improbable that the term is related to Joseph Jenkins. The conclusion would change if we find evidence that, besides writing the book on the subject, Joseph Jenkins had led some projects to popularize humanuring in Zambia in the 1980/90's; I haven't found anything to support that yet. Abecedare (talk) 17:08, 26 February 2010 (UTC)[reply]

Black holes

Every black hole will try to pull object towards it's center. Now the same force will be be from (far end) to its center at both the sides meaning that any object it pulls cannot come out from other side as it will be attracted towards the center. So then will the object will remain always at center and not come out from other end? Is there any way to measure the pull of the Black hole? —Preceding unsigned comment added by Itsrohit (talkcontribs) 18:14, 24 February 2010 (UTC)[reply]


what? —Preceding unsigned comment added by Thekiller35789 (talkcontribs) 18:22, 24 February 2010 (UTC)[reply]

I don't think this has anything to do with your question, so I've given it a new section heading. --Tango (talk) 18:29, 24 February 2010 (UTC)[reply]
See the answer provided at the section below, and in the future, please only ask the question once. — Lomn 18:48, 24 February 2010 (UTC)[reply]

is this a rigorous procedure?

what do you think of this methodology for debunking the idea of plants responding to prayer or bioenergy. I think the easiest would be something like buying a bag of potatoes, putting each one in a separate paper bag with a number on it, placing them along on a table in order (this configuration to be retained throughout, no touching them or the tables until after the experiment) then, say there are 50 numbers. Then I would with a program randomly partition each one into one of two lists, like with a random binary number, if it's 0, it goes on the first list, if it's a 1 it goes on the second list, and at the end a coin flip determines if I'll be praying and bioenergating up the first or the second list. I print the two sheets of paper twice, once to be put on the table inside an envelope but not touched again (one of each list), and the other copy is the one I use. Of the other copy, I flip a coin at the start of the experiment to see if I will be praying up and bioenergating the first or the second sheet of paper. The one I'm not bioenergating/praying up I discard. Then every day, for my remaining key, the ones that are listed I individually go to and pray and bioenergate up for 5 minutes, giving the ones that aren't on the list absolutely no love or attention. (However, I do not touch them or the table or interfere in any way, they are literally just splayed out, before they were even assigned to the first or second list).

Then, after an amount of time that is normally enough for sprouts to grow (however I don't look inside the paper bag! I still don't touch them, I simply discard the key I had been using (randomly either the first or the second sheet of paper). Then I invite a statistician student to help with my 'science project' showing him the envelope that I never touched, and telling him that it has two lists of potato numbers, one of them is the ones I fed with Miracle-Gro, and the other ones I didn't. I don't tell him anything else, but let him go to work.

If he comes back with high statistical confidence that he's determined which list is the miracle-gro group, and it matches the group I've been praying for, then uh-oh, the prayer and bioenergy must have done something, or I picked a bad statistics student :(( :((. If, on the other hand, he has no statistical confidence as to which one had been miracle-grown, then Euroka! Prayer + bioenergating debunked.

Is this a correct procedure? Thank you. —Preceding unsigned comment added by 82.113.106.92 (talk) 18:19, 24 February 2010 (UTC)[reply]

Your research student shouldn't be given a key, you just ask him to look in each bag and write down how many sprouts it has (or how long they are, or whatever). Someone that knows a bit of statistics then compares his list with you key and sees if there is a statistically significant correlation. There is no point lying to anyone about what the experiment is about. As long as the research student doesn't see the key, he can't bias the results. --Tango (talk) 18:27, 24 February 2010 (UTC)[reply]
What if I get someone else to rate each potato on a subjective scale, 1-10, for how "healthy" and "sprouty" and "energetic" it looks (third obviously very subjective). If they don't see the key, and there are enough potatoes, will it still be rigorous? For example, the statistitician then just gets a list like:

1-5/5/7 2-3/5/7 and they aren't told what the three factors are, just that the scale is 1-10. Also, there would be no evaluation at the beginning, only the end, and obviously the evaluator doesn't get to see the key. In that case, is that enough information for the statistician to say whether there is statistical confidence in differentiating the two lists based on one or several of the factors (without knowing what they are)? Thank you. 82.113.106.92 (talk) 18:38, 24 February 2010 (UTC)[reply]

One hole I see in this procedure is that since you have already decided you are "debunking" the prayer idea, your prayers are unlikely to be sincere. You should locate a person who really believes they can affect the potatoes with their bioenergy, and have them do the prayers. Hey, I'm with you on the likelihood of this one; I'm just sayin'. Comet Tuttle (talk) 18:41, 24 February 2010 (UTC)[reply]
Oh no, I have very strong prayer and bioenergy: it just doesn't do anything. 82.113.106.92 (talk) 18:45, 24 February 2010 (UTC)[reply]
Or more open-mindedly I should say: I would just like to show rigorously if it does anything to potatoes that is statistically discernible. :) 82.113.106.92 (talk) 18:46, 24 February 2010 (UTC)[reply]
By using the term "debunking" you have already invalidated the experiment. --TammyMoet (talk) 18:52, 24 February 2010 (UTC)[reply]
Actually, I think the opposite thing will happen, that there will be statistical effect from personal bioenergy. The reason I used "debunking" was so I wouldn't be called a troll/crank/all manner of other things here... :) 82.113.106.89 (talk) 19:18, 24 February 2010 (UTC)[reply]
Yes, the statistical effect will be the opposite of what you are doing precisely because you have already set your intention. I hear what you're saying about being called a crank - I'm probably the only person here who holds cranky beliefs! And it is generally held in the metaphysical community that your intention affects the outcome. Therefore, you have already set your intention and the outcome is therefore assigned. --TammyMoet (talk) 20:52, 24 February 2010 (UTC)[reply]
Such criticism only goes to show that prayer is not scientifically definable. How can you measure intent? In order to make a scientific definition of prayer, we need a set of observables - for example, a critical number of specific words must be chanted; a specific number of hand movements must be performed; etc. Any set of actions which meet that definition, whatever it is, are prayer. Since sincerity is not measurable or well-defined, it is silly to put it as a required criteria for defining some set of actions as a "valid" or "invalid" prayer. Without a rigorous operational definition for prayer, I think it is impossible to decide whether a plant has been "prayed at" or not. (How could you deal, for example, with a corner case of somebody unconnected with the experiment who has prayed that science will prove, or disprove, the efficacy of prayer? If their prayer changes the outcome of your experiment, the resulting conclusion might contradict itself.) If something as subtle as subconscious intent affects the definition of "prayer", then we need some way to measure that so that we can make the experiment repeatable. Since it is impossible to come up with a widely-acceptable objective definition of prayer, the experiment cannot be expected to attribute any measurable effect to such a hazy, ill-defined cause. Nimur (talk) 22:12, 24 February 2010 (UTC)[reply]
To maximise the Signal-to-noise ratio of your results use potatoes that have been grown by atheists (but not in holes) and do the experiment well away from churches and casinos. Cuddlyable3 (talk) 22:53, 24 February 2010 (UTC)[reply]
Hmmm - the trouble with that is that you can't tell whether atheist potatoes are responding to the prayer or benefitting from the humor of watching you do it! SteveBaker (talk) 23:37, 24 February 2010 (UTC)[reply]
Making an experiment totally rigorous is very difficult. Suppose, for example, that your mere presence near the plant makes it grow (let's say maybe the CO2 from your breath helps it out). You'd pray to half the plants and they'd do better than the ones you ignored. So that's not a rigorous experiment at all! So maybe you have to do a 'blind' experiment: Recite 'fake prayers' in front of one half and 'real prayers' in front of the other. But the experimenter might be the cause of the problem here. Maybe you get more excited when doing the 'real' praying and accidentally exhale more CO2? To fix that, you need a double-blind experiment...but that's kinda tough because you know what you're saying. Perhaps you need to find someone who doesn't understand a word of english to do the reciting - so they can't tell which random-seeming words are a "real" prayer and which are fakes. But if that fails to show a result - then maybe the problem is that the person doing the reading these words has to really believe in it. As you can see, it's exceedingly hard to get this right. HOWEVER, you can first do a very simple, poorly controlled experiment and just see if there is a massive difference to be observed. If there is, you can't make direct conclusions - but you can at least see if it's worth going to the next step. If your simple experiment shows no difference (as seems to be overwhelmingly likely in this case) - then you may not wish to proceed with the difficult stuff. SteveBaker (talk) 23:34, 24 February 2010 (UTC)[reply]
Ha! I caught Steve Baker not understanding something!!! Yay :) :) :). So, what you don't understand Steve Baker, is the meaning of "blind". You mean a placebo (fake prayers). It doesn't make sense of a potato experiment being single blinded or not, as a potato can't be told or not told whether it is the control. However, a person can be: if I am not a blind subject, I know if I am being given medicine under evaluation or just the current gold standard medicine for the piercing headache I have. I can then report my results to my researcher. In this case, the self-reported headache lengths and intensities will be skewed by the idea of having received a super-new drug being tested. (you can test this effect by giving the subjects the same old gold standard drug, but just telling one half of them that it is super new and being tested - they will not report exactly the same effects). now if the doctor doesn't tell the subject if she is getting the old or the new drug (here, fake or real prayers), then that is single-blind (which obviously isn't relevant to a potato - since they can't be told, all experiments on non-sentient objects are automatically blind). However, there is still a problem as in cases where it is very important to the researcher (say, in an aids vaccine the researcher personally helped develop) to show an effect, the researcher might consciously or unconsciously affect the behavior or reporting of the control group. Say, by just mentioning them casually a few factual pieces of information about the risks of unprotected sex with stranges, perhaps just a casual reminder that the drug is not effective and the common saying that for highly contagious diseases, from the disease's perspective, when you sleep with someone, you sleep with everyone they've slept with and so on. This could be totally subconscious, yet the brief mention could so affect the control group that they are simply disgusted by that idea; maybe they still sleep around just as much as before, but out of the pool of people they could be sleeping with, they just take ONE second to choose the ones that are less obviously slutty or shady, the ones who at least make a tiny effort to judge the other person and maybe don't say yes right away, but after 2 minutes. That SLIGHT change in behavior could, depending on the demographics and so on, TOTALLY change the epidimiology, maybe the control and test group would have both had a lot of sex, but only been infected by people who have averaged more than 100 sexual partners a year, but none of the subjects the doctor makes his careless warning to will choose the people averaging 100 sexual partners anymore, since this is pretty obvious to tell, and disgusting to them. So, if you don't want to end up with a "miracle" drug that is nothing more than a casual remark the scientist happens to make more often to one group than another (because subconsciously they don't want their test group sleeping around, increasing their risk, and "outweighing" the vaccine effect the researcher really hopes is there), then you have to make the researchers who interact with the patients have no knowledge of which medicine they are giving. They can just say: "okay, here is either the status quo drug or a candidate, I don't know which one either", and so on. This, double-blind, you can do with inanimate objects (though I guess it should just be called blinded), by not kinowing which treatment you're giving to something. How would it be possible to pray for plants without knowing whether you are currently praying or not-praying? Clearly this is impossible, and so it is not possible to make this particular experiment fully blinded... (unlike say, testing miracle-gro, in which case it is easy to make is so you yourself don't have a clue if you are putting miracle-gro or water into the soil at the moment). 82.113.106.97 (talk) 15:43, 25 February 2010 (UTC)[reply]
Eh? That's exactly my belief - except that you are assuming that the potatoes are not capable of "understanding" (well, perhaps: "reacting to") the stimulus we're testing - but that's pre-judging the result. If we're actually going to do this rather silly experiment, we have to take the possibility that it might work as a given. It's "possible" that the potatoes do indeed react to being breathed on or talked to...and if so, then that might skew the results either in favor of - or against - the hypothesis we're testing. Hence we DO have to "blind" the potatoes by randomly giving them the "gold standard" (talking to them) or the "experimental drug" (prayer). That's a blind experiment. But (as you correctly say), the "doctor" (the person who 'administers' the prayers) has to be blinded too just in case he does something to inadvertently give the potatoes what they need just because he knows which ones are supposed to "win". Everything you say applies perfectly well to what I said. I didn't misunderstand a thing. SteveBaker (talk) 03:38, 26 February 2010 (UTC)[reply]
ha-ha-ha, you're still misunderstanding it, and just made it explicit :). You say "we ... 'blind' the potatoes by randomly giving them [a or b]". That's a controlled experiment (with a test and control group). The potatoes (the test subjects) are always, automatically blinded: you can just increase it to double-blinding by not yourself (the test administrator) knowing which test you are currently administering. Here, let me give you an explanatory example near and dear to your heart: image manipulation. Specifically, image compression. I want to test my lossless compression of bitmaps against the best lossless compression in existence: it doesn't make sense to "blind" the subjects. You put the file through, then you look at the filesize, and the 'ls' (or 'dir') command can't be told or not told if it is listing the filesize of your new algorithm. Now repeat with lossy compression and a human subject, and instead of "filesize" from a cold computer program, you get feedback for a sentient subject, whether yourself or someone else. Now it's a problem if you don't blind them: if they know if they're judging your new lossy compression format or they're judging jpeg, this will affect the results. ls (or dir, or Windows Explorer, or Apple Finder) can't be told if it's telling you the filesize of your new algorithm or not. It's automatically blinded. Humans, on the other hand, very much can be told what they're evaluating. Now, double-blinding still makes sense: it means you the researcher, the one who's feeding the bitmap into the programs and then typing ls or dir and recording the declared filesize in the one case, and asking someone else or yourself about the subjective quality of the produced photos in the other case, very much can be blinded: double-blinding means you don't know if when you type "a.exe input.bmp out" and then "ls" to look at the filesize, and then typing b.exe input.bmp out" you've just tested your algorithm and then an existing one, respectively, or an existing one and then your algorithm, respectively. a and b had been randomized: one is the control, one is your algorithm, but you don't know which one until you've written down the filesizes. That's double-blinding. There's no "non-blinded" way to test the filesizes, as "ls" simply can't be told what it's looking at. HOWEVER in the lossy version of the same events it's different: the subject (unlike dir) can be unblinded "okay, here's my fantastically beautiful, more than typically accurate lossy image compression. Looking at it carefully, in your esteemed judgement ___________________ ?" and then "here is the lossy old low quality jpeg from the same bitmap source. What an awful picture. Now looking at it carefully, in your esteemed judgment ______________________ ?". That is an unblinded experiment. How can you do that with 'ls'? You can't: "look at this awfully huge file, ls, the status quo. how large would you say it is?" and then "now look at this fantastically streamlined file made with my cutting-edge algorithm, ls, how large would you say it is?" -- inanimate objects are automatically blinded, it doesn't make sense to talk about them. Now, returning to the lossy example: you can go from saying "please look at this status quo file" to saying "please look at a and tell me..." while you, the researcher, know what a is. Hopefully you won't communicate any subtle hints about whether a is the status quo or your contender, but it can happen even subconsciously. So double-blinding means that what happens is you have no idea what a and b is, and you tell your subject, "please look at a and tell me ..." and then "please look at b and tell me". Double-blinding with a computer program means you don't type "stevebakerlosslesscandidate17 input.bmp output.sb17" and then "imagemagic -whatever input.bmp output.whatever", before typing ls to compare the filesizes of "output.sb17" with "output.whatever" (the leading lossless image compression format, even if it's just zip). Instead, you type "a input.bmp output.a" and "b input.bmp output.b" without knowing whether a is a script to use your candidate program and b is a script to use the standard lossless bitmap compression algorithm, or vice-versa. Only after you've written down all the 'a' and 'b' trials with all of the source images you want to test, would you "unblind" by looking at which one had been your algorithm. In short, I hope you now understand how wrong you had been, and I want a congratulatory barn star, as this is probably the first such case. If you need more evidence, I can emend the Blind_experiment article to include it, however I hope a careful consideration will be enough for you to admit how wrong you are, and for me to get my reward money. Best regards 84.153.239.187 (talk) 14:08, 26 February 2010 (UTC)[reply]

Future citizens of Earth demonstrate (Video) a 10-fold prayer-free regime for processing Solanum tuberosum. Cuddlyable3 (talk) 18:04, 25 February 2010 (UTC)[reply]

The relavent article is efficacy of prayer. People who use prayer in non-experimental settings indicate that prayer often does "work". ~AH1(TCU) 00:38, 28 February 2010 (UTC)[reply]

Object at center of Black hole

Every black hole will try to pull object towards it's center. Now the same force will be be from (far end) to its center at both the sides meaning that any object it pulls cannot come out from other side as it will be attracted towards the center. So then will the object will remain always at center and not come out from other end? —Preceding unsigned comment added by Itsrohit (talkcontribs) 18:32, 24 February 2010 (UTC)[reply]

More or less correct. Anything that passes inside the black hole's event horizon is never passing back outside the event horizon. As for the "passing through the center" bit, that needs a little more examination. Your premise for "why" is flawed -- moving aside from black holes, there are plenty of scenarios where we can do thought experiments about dropping items through the center of some massive object. Those items don't stop right at the center even though the gravitational force is balanced, and that's because they've still got a lot of velocity. It's not until they've gone past the center of gravity that they slow down and eventually return. Back to black holes, though, it's possible that something hitting the exact center (the gravitational singularity) might instantly stop -- I'm not fully up on the weird math that takes place there. However, it's not necessarily a meaningful discussion. From an outside observer's perspective, once an item passes the event horizon, it's gone. We don't know what happens to it. Worse yet, the outside observer doesn't even observe the object pass the event horizon, due to time dilation. — Lomn 18:45, 24 February 2010 (UTC)[reply]
My understanding is that the singularity doesn't exactly experience time — that it is the future in that region of space. So things hitting it stop, but not in the normal sense because they aren't experiencing time anymore at all. --Tardis (talk) 19:09, 24 February 2010 (UTC)[reply]
Yes, if you look at the Schwarzschild metric, the radial coordinate is time-like inside the event horizon, so you cannot move outward. Icek (talk) 19:24, 24 February 2010 (UTC)[reply]

The OP refers to the black hole's "center." This is usually called the singularity. In non-rotating black holes (and probably all black holes), all objects that pass the event horizon reach the singularity, where they have already ceased to be "objects" due to spaghettification. So it is not meaningful to ask "what happens to an object" at the center, because no object remains an "object" by the time it gets there; instead, it is crushed to infinite density and simply adds to the black hole's mass. (So, to answer the OP's specific question: everything that enters eventually reaches the "center" in a different form, and "remains" there in the sense that it becomes part of the hole's mass.) This is probably also true of all black holes, but the current state of theory allows for the possibility that in some rotating black holes an object which passes the horizon might avoid the singularity; that would be a rare event in any case, and will likely be shown to be impossible as the theory progresses. Note: some information does, however, escape from black holes, though "objects" as such do not (see Black hole information paradox). 63.17.57.56 (talk) 03:44, 25 February 2010 (UTC)[reply]

"Crushed to infinite density" doesn't make sense - when an infinity appears in physics like that it is mathematics way of telling us we've made a mistake. Something happens at the centre of a black hole, but we don't know what. --Tango (talk) 17:37, 25 February 2010 (UTC)[reply]

Supplementary question: When an object is attracted towards a black hole is the black hole attracted towards the object? When the object accelerates towards the black hole are the Conservation Laws of momentum and energy conserved by proportional movement of the black hole? Cuddlyable3 (talk) 17:33, 25 February 2010 (UTC)[reply]

Yes. At large enough distances, a black hole behaves just like any other object of that mass. --Tango (talk) 17:37, 25 February 2010 (UTC)[reply]
What is that mass? Cuddlyable3 (talk) 19:27, 25 February 2010 (UTC)[reply]
It may be any mass. There is no theoretical limit on how heavy or how light a black hole may be. Dauto (talk) 21:41, 25 February 2010 (UTC)[reply]
If 1gm of material falls into a black hole does the mass of the black hole increase by 1gm? Can a black hole's mass be as small as a subatomic particle? Cuddlyable3 (talk) 22:56, 25 February 2010 (UTC)[reply]
Yes. Maybe. Really small black holes evaporate in a fraction of a second by Hawking radiation. Working out exactly what happens to the smallest black holes would require a theory of quantum gravity, which we're still working on. --Tango (talk) 03:26, 26 February 2010 (UTC)[reply]
Tango states that "'Crushed to infinite density' doesn't make sense." Neither does the quantum leap (how can something go from one place to another without crossing the intermediate area?), but it's real. When the overwhelming consensus is "A," simple standards of citation and authority make "B" incorrect; SO, it is incorrect to say that the singularity does not have infinite density. It may eventually be shown that it doesn't, but as of the last 40-plus years (since it was demonstrated by Penrose), it has been the overwhelming consensus of all citable authority that the singularity has infinite density. Simply google "black hole singularity" and you'll find dozens of authoritative sources stating exactly that. Tango writes "when an infinity appears in physics like that it is mathematics way of telling us we've made a mistake," but this is an opinion supported by no citable authority and actually disproven to the extent that general relativity describes the final stages of massive stars. "A black hole is what you get if you compress so much mass into a region of space that it collapses, under its own weight, to an infinitely small, dense, point called the 'singularity.'" -- Leonard Susskind at http://calitreview.com/790 . See also http://www.scribd.com/doc/9517358/Michael-Good-The-Black-Hole-Singularity ... and about eleventy zillion other sources. Yes, it's true we don't know what happens in the singularity -- and in fact it is unobservable by definition, so any theory will be strictly mathematical. But the singularity has infinite density, whether or not that offends someone's sense of colloquial or semantic logic. —Preceding unsigned comment added by 63.17.46.122 (talk) 03:35, 26 February 2010 (UTC)[reply]
There may be variants of a black hole's "center". A ring singularity, for example. ~AH1(TCU) 00:15, 28 February 2010 (UTC)[reply]

Liquid-mixing scintillation effect

Sometimes when two miscible liquids are mixed, you can see a hard-to-describe scintillation effect (sort of like you can see the two liquids mixing, even when they're the same color). It works when pouring cold water into a cup of hot water, if that helps. What is this called, and what causes it? 76.204.127.175 (talk) 21:28, 24 February 2010 (UTC)[reply]

It's caused by the two liquids having different refractive indexes: light bends "differently" in each. See Schlieren photography. DMacks (talk) 21:44, 24 February 2010 (UTC)[reply]

Subject catgeorization

The University of Edinburgh Library has shelved some books in strange categories. When this was pointed out they gave the excuse that sometimes books have more than one possible category. That may be a valid reason sometimes but I don't think so in this case. I can't figure out why these books have been categorized in the way they have so I'm asking here in the hope that someone will be able to explain what these books have to do with the subjects they have been put in. "Measure, integration, and functional analysis‎" by Robert B. Ash was in the geophysics section, as was "The elementary process of Bremsstrahlung" By Eberhard Haug, Werner Nakel, and a book on the evolution of binary stars was also in the geophysics section. The book "Quantum Field Theory on Curved Spacetimes" by Christian Bär, Klaus Fredenhagen was in the biophysics section inbetween a book on zoological physics and a book on insect populations. I took these books to the library desk to enquire about this. The next day the books had been put back in the same place. Upon taking the "Quantum Field Theory" book back to the desk, I was told that the librarian had checked and this book has been categorized the same way in other places as well. I reckon the librarian is talking bs but I would love to be proved wrong. —Preceding unsigned comment added by 217.44.34.55 (talk) 21:53, 24 February 2010 (UTC)[reply]

Somebody, somewhere, has clearly screwed up. It may well not be at that library, though. Library often get their classifications from some central source. If that central source has screwed up, then the librarian could well be right that other places have the same problem. That doesn't mean the librarian shouldn't do something about it, though... Does the physics department have a designated library liaison? If so, it might be better if they talked to the librarian about the problem. --Tango (talk) 22:08, 24 February 2010 (UTC)[reply]
See the article on Dewey Decimal Classification in case the library uses that system of categories. You may find the librarian to be more receptive if you can point out the correct categorisation in Dewey terms. Cuddlyable3 (talk) 22:38, 24 February 2010 (UTC)[reply]
Looks like the University of Edinburgh Library uses the Library of Congress classification. 75.41.110.200 (talk) 23:23, 24 February 2010 (UTC)[reply]
I don't know why libraries have so much trouble over such a relatively simple data organisation problem. If you have all your books listed on the Internet along with their shelved position - then the library can be classified in any and all ways imaginable - books can be in multiple categories without problems. The only reason shelf order matters in an era of near-universal computer access is for people who browse looking for books in a general category. But even that is well handled by things like "Other people who borrowed this book also borrowed..." kinds of search criteria. The biggest problem is when a book gets put back in the wrong place by accident - so if I were organizing a system, I'd put computer-generated colored stickers on the spines so that all of the books on one shelf should have the same combination of colored stickers - if a book is on the wrong shelf, you can spot it in seconds. SteveBaker (talk) 23:22, 24 February 2010 (UTC)[reply]
First, I have spent a long time working on the "Other people who XXX this..." algorithms and I think a proper algorithm would be "Other people who borrowed this book then borrowed...". Time ordering plays a role in which books you may want to read. Second, There are libraries that use color coded stickers. I've seen many (ie: the public library in Charleston County, SC). I've also seen work on using barcodes. A robot can scan shelves very quickly at night and take note of all incorrectly shelved books. It would be optimal if it could also correct shelving. Finally, as for categorizing books. Some libraries shelve books in multiple locations. It was difficult to manage when it was all done by hand, but computers make it easy to manage multiple copies of a book sitting in multiple locations (even multiple libraries). —Preceding unsigned comment added by Kainaw (talkcontribs)

February 25

salt in water

since i got my wisdom teeth out recently ive been drinking warm water in half a glass with half a teaspoon of salt to clean the mouth. anyway today i heated up the water for 30 secs in the microwave, then put the half teaspoon of salt in the glass. i then stirred the glass with the teaspoon. as i was stirring, the spoon made a low scrapping sound, and as the salt dissolved, the sound grew louder. when all the salt was dissolved, stirring the glass sounded normal.

i was wondering what was happening with the sound. was the salt absorbing the sound? it was really cool.--Minor330 (talk) 01:16, 25 February 2010 (UTC)[reply]

I would think it would be perfectly obvious: so obvious in fact that I will resist the urge to explain. I think you are testing the reference desk to see just how stupid people can be. But if you are not, well, the sound ceased when the salt disappeared, because the salt was making sound as it scraped the glass. Come on. Vranak (talk) 01:52, 25 February 2010 (UTC)[reply]
Is that really an appropriate tone to take in answering a question that appears to have been asked in good faith? Particularly since you seem to have missed the key part: that the sound got louder as the salt dissolved, which is what the OP is querying. Possibly related to the change in pitch as you stir a hot chocolate? 131.111.248.99 (talk) 02:16, 25 February 2010 (UTC)[reply]
We had a very similar question before - (someone should check the archives - I had a quick search but couldn't find it) I'm 99% sure the salt and the heat has nothing to do with it. What I think makes a big difference is that when the liquid is already swirling it makes a dramatically different sound than when it's stopped or moving slowly without turbulence. SteveBaker (talk) 03:59, 25 February 2010 (UTC)[reply]
This question was similar. Cuddlyable3 (talk) 17:24, 25 February 2010 (UTC)[reply]
Yeah - that's the one I was thinking of. Thanks! SteveBaker (talk) 03:19, 26 February 2010 (UTC)[reply]
Good faith has its limits. Even so I have accounted for the possibility that the questioner did not know what was happening even though he already explained exactly what was happening. Groan. Vranak (talk) 04:00, 25 February 2010 (UTC)[reply]
How do you propose to explain why the sound grew louder as the salt dissolved? If the salt was responsible, shouldn't the sound become quieter? --99.237.234.104 (talk) 04:54, 25 February 2010 (UTC)[reply]
Related : The Straight Dope : When I stir my coffee, why does the sound gradually change pitch? APL (talk) 15:41, 25 February 2010 (UTC)[reply]
I would surmise that it is due to the changing speed of sound in different strata. Vranak (talk) 18:49, 25 February 2010 (UTC)[reply]
After the last time we were asked this (Thanks Cuddlyable!) our OP claimed that (s)he could reliably "reset" the sound by stirring the liquid. If that's true, it busts the dissolved gasses theory - so I think "The Straight Dope" guy is guessing. SteveBaker (talk) 03:27, 26 February 2010 (UTC)[reply]

Chemical looks.

I have six out of: 1-aminobutane, benzaldehyde (according to wikipedia - colourless liquid), benzoic acid (according to wikipedia - colourless solid), butanone (according to wikipedia - colourless liquid), benzophenone (according to wikipedia - white solid), ethanamide, ethylbenzene carboxylate and benzonitrile. Two of my substances are white flakey powder. Two are clear liquids. One is a clear solid in small wet-ish picces like powder. One is a slightly yellowy liquid. Could you please help work out the which would be which? Particularly which the clear solid is and the yellowy liquid is? Also, one smells absolutely vile, which is it? All help is greatly appreciated. —Preceding unsigned comment added by 81.157.96.6 (talk) 02:00, 25 February 2010 (UTC)[reply]

Please do your own homework.
Welcome to the Wikipedia Reference Desk. Your question appears to be a homework question. I apologize if this is a misinterpretation, but it is our aim here not to do people's homework for them, but to merely aid them in doing it themselves. Letting someone else do your homework does not help you learn nearly as much as doing it yourself. Please attempt to solve the problem or answer the question yourself first. If you need help with a specific part of your homework, feel free to tell us where you are stuck and ask for help. If you need help grasping the concept of a problem, by all means let us know. --ColinFine (talk) 08:59, 25 February 2010 (UTC)[reply]
Sorry, I forgot my login details before.
It's not a homework question but a question for mock coursework practical. We're allowed to do as much research as we want and take in notes, so I've tried to find out elsewhere but I can't; this is far from my first port of call. It would be extremely useful to find out which the yellowy liquid is. Thank you for any help you could possibly provide. Chocolate muffins in a basket 11:12, 25 February 2010 (UTC)[reply]
Right, I strongly suspect the stinky stuff is 1-aminobutane (there's an article here on n-butylamine, I might put in some redirects later so it's all IUPAC'd up). Amines in general are fishy or ammoniacal as far as smell goes. It should also be the slightly yellow liquid. You could probably work out which one is benzoic acid with universal indicator (or, to show off, an acid-base titration). As for the others, I'm sure you can come up with something. Brammers (talk) 11:26, 25 February 2010 (UTC)[reply]

Thank you very much. I've looked at previous experiments and it looks like it is 1-aminobutane that smells. I also have which is benzoic acid. And I can easily work out the other two solids using Brady's reagent. Thanks. Chocolate muffins in a basket 11:33, 25 February 2010 (UTC)[reply]

You're welcome! Also, just remembered that benzaldehyde smells strongly of almonds, which could be handy too. Good luck with the coursework! Brammers (talk) 16:29, 25 February 2010 (UTC)[reply]

Nonrenewable Energy

Coal, oil, natural gas, and nuclear fuels such as uranium are nonrenewable. So if we continue to use them at the current rate, when will they be used up?

An Unknown Person (talk) 09:13, 25 February 2010 (UTC)[reply]

Our fossil fuel article gives the estimates below.
Years of production left in the ground with the most optimistic proved reserve estimates (Oil & Gas Journal, World Oil)
  • Oil: 43 years (or 43 years using proven levels and flows in the fossil fuel article)
  • Gas: 167 years (or 61 years using proven levels and flows in the fossil fuel article)
  • Coal: 417 years (or 148 years using proven levels and flows in the fossil fuel article)
but it's a big assumption to say we continue to use them at the current rate. Currently our use is growing, but Hubbert peak theory claims that use will decline as reserves run out and prices get higher.
Estimating how long uranium reserves will last is also tricky, because it depends on whether breeder reactors are used (and because the industry is currently experiencing a regrowth). World energy resources and consumption gives an estimate of 70 years and then says most industry observers disagree (which is helpful). AlmostReadytoFly (talk) 11:38, 25 February 2010 (UTC)[reply]
The French authority NEA gave 100 years based on 2006 consumption rates.[4] AlmostReadytoFly (talk) 11:48, 25 February 2010 (UTC)[reply]
This link World_energy_resources_and_consumption#Nuclear_fission gives the reserves of nuclear fuel as 2500ZJ. With a worldwide total energy consumption of 474EJ (your first link) I calculate more than 5000 years reserves of nuclear fuel even if we would stop using any other energy sources completely. 95.115.141.196 (talk) 13:40, 25 February 2010 (UTC)[reply]
Does this presume 100% energy recovery from the 2500ZJ,or is that already factored in? Googlemeister (talk) 17:29, 25 February 2010 (UTC)[reply]
Dunno, that's just what the article says. Let it be pathetic 10% and it still will be enough for a good 500 years. (Don't get me wrong, I'm not into soapboxing today (I said: todoay) and not saying anything about nuclear waste.) 95.115.141.196 (talk) 18:06, 25 February 2010 (UTC)[reply]
For the fossil fuels (coal, oil, gas), we can't run out because if we burned all that there is, the greenhouse gasses produced would make the earth virtually uninhabitable. Uranium is a much bigger unknown. If we manage to get fusion working reasonably - then there are truly vast reserves of hydrogen to work with...although a lot depends on the final technology. While you're worrying, you might want to consider the world reserves of copper, helium and the "rare earth" stuff from which high powered magnets are made. All three will run out long before fossil fuels - and at least copper and rare-earth magnets are essential to the very energy reduction efforts in things like electric cars - and in renewables such as wind turbines. Several other rare metals that are frequently used in batteries and high-tech electronics come only from single sources in places that are politically unstable. Not good! SteveBaker (talk) 14:14, 25 February 2010 (UTC)[reply]
From Rare earth element: "rare earth elements are found in relatively high concentrations in the earth's crust". 95.115.141.196 (talk) 14:51, 25 February 2010 (UTC)[reply]
They are called "rare" for a reason. These concentrations aren't generally high enough to be economically viable. See this, for example or this. SteveBaker (talk) 03:08, 26 February 2010 (UTC)[reply]
So the problem is not the world reserves as such but the ore quality. It takes more energy to concentrate and extract the metals. So it boils down to the availability of energy. Is China politically unstable? 95.112.185.232 (talk) 08:47, 26 February 2010 (UTC)[reply]
I clearly remember when I was still at school (about 1976?) reading an earnest newspaper report saying that the world's oil reserves would be exhausted in 1984. Prediction is not a precise science. Alansplodge (talk) 17:32, 25 February 2010 (UTC)[reply]
There was a very obese American pundit who around that time was well known for saying that the world would get increasingly materially wealthy. Who was he, and was he right? 78.147.93.182 (talk) 21:21, 25 February 2010 (UTC)[reply]
We could use all the fossil fuels very slowly without global warming being an issue. The half-life of CO2 in Earth's atmosphere is, perhaps, a few thousand years (it is unclear), so if we take 50,000 years to use it all the fossil fuels, we should be ok. --Tango (talk) 03:23, 26 February 2010 (UTC)[reply]
We've had 40 years worth of oil left for at least the last 50 years. Take all such estimates with a tablespoon of salt. They are usually based on known reserves accessible with current technology, so are just a lower bound. As we search more and our technology improves, we find more oil we can access. We will never actually run out, of course, since as we get near the price will go up and people will switch to alternatives that will be cheaper. --Tango (talk) 03:23, 26 February 2010 (UTC)[reply]
The price is already going up. Haven't you been living in this planet the last few years? Dauto (talk) 03:44, 26 February 2010 (UTC)[reply]

moon mission

why is it that no other country other than us has been able to sent a man on moon. while us had done it almost 50 years ago. do all other countries cant create such a mission even after so many years.....piyush (talk) 09:15, 25 February 2010 (UTC)[reply]

They probably didn't think it was worth the cost. Watch an expert explain it.
Ben (talk) 09:44, 25 February 2010 (UTC)[reply]
There was a lot more prestige involved in a Cold War space race. Clarityfiend (talk) 10:17, 25 February 2010 (UTC)[reply]
Could it be done at all today? 95.115.141.196 (talk) 11:53, 25 February 2010 (UTC)[reply]
It is worth mentioning that some people think it didnt happen Moon_landing_conspiracy_theories. If it did happen 50 years ago, it should be orders of magnitude cheaper and easier to do now, due to advances in technology and industrial processes. Therefore, a billionaire could easily do it himself. —Preceding unsigned comment added by 194.196.95.89 (talk) 11:58, 25 February 2010 (UTC)[reply]
Well, this is not a place for soapboxing, so I leave it like that. 95.115.141.196 (talk) 12:16, 25 February 2010 (UTC)[reply]
Computers have been getting cheaper because the components have shrinking and mass production has been encouraging research, (and probably because computation is a fundamentally easy thing to do, with the laws of physics we have). Even if we were mass-producing moon rockets, that wouldn't change the fact that the only way to get to the moon is to strap yourself to a thirty-story tower of explosives. (the Space Shuttle has a comparatively easy job, since it only goes to low earth orbit.) Paul Stansifer 14:35, 25 February 2010 (UTC)[reply]
The reason other countries didn't bother is the same reason why the US stopped going there and has never returned...there was prestige value to being the first (and the US was really desperate for the Soviet Union not to get that prestige) - there was (and still is) no prestige value in being second. Aside from that - there is little reason to go to the moon. Until we have the technology to create robotic factories that could do something interesting with the moon's resources, it's essentially an entirely boring place. With robotic factory technologies, there are a few interesting projects:
  1. Build massive solar power generation facilities at the lunar poles and beam power back to earth from them. (Although, arguably, doing that at a suitable Lagrange point might be easier).
  2. Build a telescope on the back side of the moon where it could see without the earth being in the way - and a radio telescope would not pick up all of the radio pollution from here on Earth.
  3. Mine Helium3 and ship it back to Earth to make it easier for us to build fusion power plants.
  4. Use solar power and water from deep craters near the poles - make hydrogen and oxygen for deep space missions - and to provide for an eventual human colony on the moon.
But none of those things could usefully be done by a handful of people going for a few days as in the Apollo era. It takes massive construction equipment - and that means either a large group of humans (which would be ruinously expensive) or fancy autonomous robots (which we don't yet have the technology to provide).
So there is literally no reason to go there. It's a very dead, boring place. We have moon rocks to study and we have really detailed photos...what more do we need? 14:05, 25 February 2010 (UTC)

A French expedition to the Moon found it occupied by an alien civilisation. Cuddlyable3 (talk) 17:18, 25 February 2010 (UTC)[reply]

Hmmm - good point. A very dead, boring place...with Frenchman - that's an even better reason not to go there! :-) SteveBaker (talk) 19:30, 25 February 2010 (UTC)[reply]
50 000 000 != wrong. Cuddlyable3 (talk) 20:18, 25 February 2010 (UTC)[reply]
It is important to distinguish between not being able to go to the Moon and not trying to go the Moon. The US is the only country to ever fail to land on the Moon (see Apollo 13). --Tango (talk) 01:37, 26 February 2010 (UTC)[reply]
So they say... ;-) --Mr.98 (talk) 04:20, 26 February 2010 (UTC)[reply]
By that logic Tango, I am the greatest professional baseball pitcher ever, because not only have I never allowed a run, I have never walked a batter, given up a hit, or even failed to get a strike on every pitch! A sample size 0 offers no data whatsoever. Googlemeister (talk) 19:22, 26 February 2010 (UTC)[reply]
The truth is getting to the moon was exceptionally difficult in 1969 and had not a huge amount of payoff other than propaganda. Could the US do it again? Probably, but it would take a lot of specialized work to do it. Sending up satellites and even ISS missions is a very different sort of work than landing on the moon. You don't immediately go from sending unmanned probes to making manned craft again in a short working order. A space shuttle can get people and things into space and back again (with still some difficulty), but it is not the same thing as landing on a totally different body with different gravity and etc. It's a non-trivial task, even today. The reason no one else has bothered is because there is great risk, great cost, little reward. --Mr.98 (talk) 04:20, 26 February 2010 (UTC)[reply]

Steam

I was given two questions, to which I'm pretty sure I know the answer to, but I would like to confirm them.

Q1: "When given a bowl of chicken soup, which way does the smoke go, and why?" My answer would be that it travels upwards, because the water vapour rising off the surface of the bowl is hotter, and therefore less dense, than the surrounding air. However, I have an auxillary question (this is my own): would the fact that this steam has more water vapour significantly affect it's relative density?

Q2: "When a cold box is opened, which way does the smoke go, and why?" My answer would be that it goes downwards, because the vapour is now colder than the surrounding air. Auxillary question: why is there vapour in the first place? Is it because air trapped inside the box when it's closed gets cooled down, which causes the water in the air to condense? —Preceding unsigned comment added by 173.179.59.66 (talk) 09:44, 25 February 2010 (UTC)[reply]

First vapor is not smoke. What you see when you open a cold box, vapor apparently going out, is in reality cold air condensing water vapor that was outside the box. Non Zero-sum Ed (talk) 11:33, 25 February 2010 (UTC)[reply]
Q1: See the article Steam. What you see as smoke consists of hot water vapour (invisible) plus a visible white mist of water droplets where the vapour condenses in contact with external cold air. The mist is an Aerosol that is slightly denser than air and eventually falls as it cools.
Q2: What you see as smoke is invisible cold air plus a visible white mist of water droplets where external water vapour condenses in contact with the cold air. Otherwise your answers are correct. Cuddlyable3 (talk) 16:41, 25 February 2010 (UTC)[reply]

So, to confirm: in Q1, the smoke will first rise, due to higher temperatures, but then fall as it cools due to higher density; in Q2, the smoke will fall due to the combination of being both cooler and denser than the surrounding air? —Preceding unsigned comment added by 173.179.59.66 (talk) 05:13, 26 February 2010 (UTC)[reply]

Donating plasma

According to the American Red Cross [5], referring to plasma: The "best blood type to donate: AB+, AB-, A+, A-" Why are these blood types better?--Non Zero-sum Ed (talk) 11:30, 25 February 2010 (UTC)[reply]

According to blood type "Type O plasma, containing both anti-A and anti-B antibodies, can only be given to O recipients. The antibodies will attack the antigens on any other blood type. Conversely, AB plasma can be given to patients of any ABO blood group due to not containing any anti-A or anti-B antibodies." This indicates why AB is good and O is less useful. I'm not sure why they prefer A over B --Normansmithy (talk) 12:00, 25 February 2010 (UTC).[reply]
According to blood type type A is more common - therefore type A plasma can be used to transfuse more peoplem and is therefore more useful. --Phil Holmes (talk) 12:15, 25 February 2010 (UTC)[reply]
In my fantasy world, where everyone who can would donate some blood product, I would want especially O to donate whole blood rather than "just" plasma, since universal-donor is particularly useful itself (as opposed to "O plasma is not as useful as others'"). Totally speaking from my own head only here. DMacks (talk) 21:10, 25 February 2010 (UTC)[reply]

milk ph

milk is a basic ph but i heard when it is drunk it is converted by the body to a acid, is this true? is drinking milk beneficial or detrimental to raising the bodys ph ? —Preceding unsigned comment added by 67.246.254.35 (talk) 15:40, 25 February 2010 (UTC)[reply]

You are probably talking about lactic acid. Some are affected more by it than others. It isn't really beneficial or detrimental to the body's ph level. -- kainaw 15:46, 25 February 2010 (UTC)[reply]

im not referring to lactic acid —Preceding unsigned comment added by 67.246.254.35 (talk) 16:26, 25 February 2010 (UTC)[reply]

Milk isn't basic pH, it's slightly acidic at around 6.4-6.8. I doubt it would have either a detrimental or beneficial effect on the body with respect to the pH. Someone feel free to prove me wrong. Regards, --—Cyclonenim | Chat  17:53, 25 February 2010 (UTC)[reply]
The body is strongly buffered. I don't think you could drink enough milk, even if it were converted to acid, to change the body's pH. As an aside, acid lowers pH (bases raise it). DMacks (talk) 18:03, 25 February 2010 (UTC)[reply]
We are mammals. We've evolved to drink milk. We're capable of spending the first year of our lives living on nothing else whatever! If you have the gene for lactose tolerance (as a good majority of humans do) then your digestive system should work just great with stuff that has whatever pH milk has. Why would you think otherwise? SteveBaker (talk) 19:27, 25 February 2010 (UTC)[reply]
Actually Steve, a good minority of humanity has the gene for lactose tolerance; the majority become lactose intolerant as they grow up, like most mammals. A good majority of European humans have the gene, relatively recently evolved, for lactose tolerance. 86.177.121.239 (talk) 20:04, 25 February 2010 (UTC)[reply]


yes i understand milk is about 6.8 ph but what i heard is that for some reason the body coverts it to about 4.0 or lower is this true? —Preceding unsigned comment added by Thekiller35789 (talkcontribs) 19:32, 25 February 2010 (UTC)[reply]

I still don't get where you thought milk was basic then, if you understand that milk is slightly acidic and you thought it went even more acidic. Anyway, no, it won't be as low as 4.0 because all that is happening during digestion is the bonds are breaking and new products are forming, mostly protein and lipids. Neither would be at pH 4.0, I don't think. Regards, --—Cyclonenim | Chat  20:55, 25 February 2010 (UTC)[reply]
I was always told milk was basic when I was younger, and that it can be used to neutralize battery acid if a battery leaks. It wasn't until later that I found out that it was acidic. Falconusp t c 02:47, 26 February 2010 (UTC)[reply]
Yes, I used to think milk was alkali... anyone know where that misconception comes from? --Tango (talk) 03:02, 26 February 2010 (UTC)[reply]
Milk is buffered, which is a fancy chemical way for saying that it resists changes in pH. Essentially, regardless of whether you add an acid or a base to milk, it tends to rebound back to near its native pH level, which is how buffers work. Add a base, and the pH snaps back to the pH of milk. Add an acid, and the pH snaps back. Which is why people have vague recollections of being told to drink milk to treat either acid or base poisoning. Also, the buffering effect of milk is why drinking too much milk can make you throw up. The classic "you can't drink an entire gallon of milk in one hour" is because your stomach expects a very low pH. There are biofeedback mechanisms which, if your stomach pH is out of whack, causes you to throw up. Because milk is a buffered solution, when you drink a lot of it, it causes your stomach to think you've ingested something which is bad for you, and causes your vomit reflex to kick in. You can easily drink a gallon of plain unbuffered water in the same time period, because it isn't buffered, and so will not drastically change your stomach's pH as much as milk will; even though pure water is at a more basic pH than milk. --Jayron32 05:41, 26 February 2010 (UTC)[reply]
Fascinating - thank you. --Tango (talk) 06:15, 26 February 2010 (UTC)[reply]

Articles about plants and animals

Well, this is a question that might sound like a complaint. If so, it's against the world as such, not against the contributors of wikipedia. And it's hard to put it in a question.

I was looking at Jerusalem artichoke and would have liked to know plants that grow in similar climate/soil. No way to search for them before I know the names already?

I was looking at Rock samphire and would have liked to know how it manages to grow practically on stones.

When looking at articles about animals I'm generally not much interested in their dental formulas but in their ecological niches and, for example, what a lynx does better or worse than a competing fox in the same wood.

Am I somehow perverted to be interested in such things, or why is wikipedia lacking such information? 95.115.141.196 (talk) 16:25, 25 February 2010 (UTC)[reply]

Curiosity is healthy and Wikipedia is still a work in progress with room for improvement. Cuddlyable3 (talk) 16:45, 25 February 2010 (UTC)[reply]
The questions are great and when I have similar questions, and I don't have the resources to find the answers, I go to the "discussion" page of the article and add a new section asking for a knowledgeable editor to add a section with the information. Some day someone may do it. Comet Tuttle (talk) 18:00, 25 February 2010 (UTC)[reply]
I agree entirely with that - but people should recognize that there are limits to the depth of information that an encyclopedia can offer. I absolutely guarantee that the information that's being sought here isn't in (say) Encyclopedia Britannica. There comes a point when searching for very deep knowledge when you have to say "Well, I need to get a proper textbook on that subject". If we wanted to say everything there was to say about (say) Foxes - then the article would probably be 1000 pages long - about the length of a handful of books and a few dozen scientific papers about the animal. There is little likelyhood of that much information being written about every single animal, plant and Japanese railway station in existence (well, maybe there is a chance for the railway stations!). SteveBaker (talk) 19:25, 25 February 2010 (UTC)[reply]
I agree that an article cannot contain everything. But have you had a look at Rock samphire? Can you tell me seven articles about animals that refer to their ecological niche in comparison with their competitors? In the previous millennium (exaggeration intentional) Encyclopedia Britannica and dental formulas were the state of art, and a hand full of people making celluloid pictures of African animal wildlife became the avant-garde of zoology. (And finally I can't avoid soapboxing: we have fallen far behind our possibilities, in many aspects; pluck lost, all lost.) 95.115.141.196 (talk) 19:48, 25 February 2010 (UTC)[reply]
Steve Baker is conscious of the Information explosion. But Wikipedia has presently 3 200 000+ articles (English version), it's still growing and we are told WP:DWAP. Moore's law seems eternal so what can stop Wikipedia ìncluding everything? Software bloat? Search engine overload? Vandalism? Editors departing to jobs that pay real Money? Cuddlyable3 (talk) 20:42, 25 February 2010 (UTC)[reply]
There's more between client and server than a linear search algorithm could imagine. Disk space lost, little lost. Information lost, much lost. Pluck lost, all lost. 95.115.141.196 (talk) 21:06, 25 February 2010 (UTC)[reply]
What the pluck? Cuddlyable3 (talk) 22:48, 25 February 2010 (UTC)[reply]
I've always thought that a free online database and Wiki about garden flowers (and vegetables) would be very useful. There must be around twenty different pieces of information that a gardener would like to know when choosing flowers to grow in their garden. Even the best gardening books are incomplete and only give some of the information you need. 78.151.155.128 (talk) 01:00, 26 February 2010 (UTC)[reply]
There are lots of kinds of information that I'd love to have in Wikipedia too - but wishing it were here and actually getting it done is a very different matter. Category:Garden_plants links to about 300 articles about garden plants, List_of_garden_plants appears to link to several hundred also. Researching, referencing and adding all of that information would a very considerable effort. If someone wanted to undertake that, I doubt there would be any objections to creating an infobox to collect the relevant stuff in a uniform manner. There are a lot of subjects that would benefit from that treatment - but it takes some dedicated enthusiasm to make it happen.
I often joke about articles about Japanese Railway stations - but just marvel at List of railway stations in Japan - there are by far more articles about Japanese railway stations than about garden plants. Each one has a photo, a list of other stations on the same line, the history of the place, what bus stops there are there. It's crazy! I doubt that those articles get one visitor a year each! There are articles about all of the railroad companies, all of the rolling stock - you name it, if it's anything remotely to do with Japanese railways - and there is a really good Wikipedia article on it! Wikipedia:WikiProject_Trains_in_Japan is an entire user group set up entirely to create and administer these articles and standards for quality for them.
I completely agree that keeping all of that information about what kinds of soil, light, watering and feeding each kind of garden plant needs is vastly more useful than having 500 articles about Japanese railway stations in an English-speaking encyclopedia. But it's a labor of love. A small group of editors does that (and probably very little else) and it's an incredibly detailed record. If some person (or small group) were to attack the garden plant articles with the same enthusiasm as the Japanese railway folks - the encyclopedia would be better for doing that. But we can't possibly do anything to make it happen.
If someone with the knowledge and enthusiasm comes along, we'll get it. If not...not. But again, compare us to other encyclopedias. Aside from specialist gardening encyclopedias - I doubt most general purpose encyclopedias have half the coverage we have at half the depth we have.
SteveBaker (talk) 02:54, 26 February 2010 (UTC)[reply]
The information about flowers would get really useful when you can search all of it. The BBC has an online database of flowers which is the best I've seen anywhere but only some of the information is searchable - you cannot for example search for those flowers that should be sown in february. I wonder if it would be possible to use existing AI to search through all the existing flower articles in Wikipedia and construct a database automatically? 92.29.32.229 (talk) 21:07, 26 February 2010 (UTC) 92.29.32.229 (talk) 21:04, 26 February 2010 (UTC)[reply]
One of the directions I'd passionately like to see Wikipedia diversify into would be to start a formalization of the knowledge that Wikipedia holds into something like the Cyc representation. This is a formal language that represents raw factual knowledge in a form that computers can readily understand. There have already been some some investigations into this possibility. Some examples from our article are:
(#$isa #$BillClinton #$UnitedStatesPresident)  --"Bill Clinton belongs to the collection of U.S. presidents"
(#$genls #$Tree-ThePlant #$Plant)  -- "All trees are plants".
(#$capitalCity #$France #$Paris)   -- "Paris is the capital of France."
(#$relationAllExists #$biologicalMother #$ChordataPhylum #$FemaleAnimal)  -- All objects that are Chordata have a mother who is a female animal.
Cyc systems use relatively simple inference systems to answer complicated questions using this kind of database: "Did all United States Presidents have mothers?". Cyc could check that United States presidents are required to be human and that humans are Cordata's and therefore, yes, they do - but then it could check all of our articles about US presidents and see if each one has an explicit mention of a mother - thereby causing us to be notified that there are three articles about US presidents whose mothers are not mentioned. It would be wonderful to have the ability for software to automatically extract knowledge from the encyclopedia - as well as do complicated things like check it for consistency and automatically find references from one article that could be used to verify facts in others. Our existing system of links and categories could become something much more intelligent. It could easily replace the "Category" system with something completely automatic - because "Category:UnitedStatesPresident" would automatically be populated with people for whom there is a "(#$isa #$xxxxx #$UnitedStatesPresident)" entry. Obviously we'd want to wrap this horrible syntax with a nice GUI that lets you enter facts in this way.
The power of having the whole of human knowledge stored in a computer-understandable way would be phenomenal. I think that such a thing would have as big an impact on our society as the Internet itself. SteveBaker (talk) 22:04, 26 February 2010 (UTC)[reply]
Ah, that's the kind of answer I wanted to see. How can we do this? How can I help? 95.112.185.232 (talk) 22:29, 26 February 2010 (UTC)[reply]
We are closer than you might think. See DBpedia. --Stephan Schulz (talk) 22:31, 26 February 2010 (UTC)[reply]

Mood Rings

Please explain in layman's terms how the stone on the mood ring changes color. Thanks --12.170.106.12 (talk) 18:28, 25 February 2010 (UTC)[reply]

Have you tried reading Mood ring? Is there something specific you don't understand? --Tango (talk) 18:34, 25 February 2010 (UTC)[reply]

It said something to the type of crystal it is but how does this crystal change color? What makes it different say when you touch a quartz crystal? --12.170.106.12 (talk) 18:36, 25 February 2010 (UTC)[reply]

The Liquid crystal article explains this in a reasonable amount of depth. APL (talk) 18:50, 25 February 2010 (UTC)[reply]
They use the same principle as one of those plastic-strip thermometers - it uses a liquid crystal that changes color as a function of temperature. The theory (and it's a terrible one) for how they tell your "mood" is that your body temperature is supposed to change when you get angry, etc. The trouble is that to the extent that this is true (which is to say "not much") is far FAR less than the color changing chemical will require to change color. By far the most significant thing that determines the temperature (and hence color) of the ring is how warm the air is around it. Proponents of these pieces of junk claim that in women, their "time of the month" affects their body temperature - and also their mood - but even that is really hard to justify. Basically, mood rings are junk. SteveBaker (talk) 19:11, 25 February 2010 (UTC)[reply]
At least they're less gaudy than a hypercolor shirt. APL (talk) 20:06, 25 February 2010 (UTC)[reply]
I hear they are really good at telling you the mood of your pet rock though. SteveBaker (talk) 02:27, 26 February 2010 (UTC)[reply]
Ah! Then they're obsolete! Modern pet rocks are USB compatable. APL (talk) 06:14, 26 February 2010 (UTC)[reply]

Killing those fun guys.

Fungi are annoying, often pathogenic, and because they're eukaryotes they're a bitch to treat. Therefore, when treating, presumably we have to develop drugs which primarily disrupt a fungus' unique characteristics. One such characteristic would be the presence of hyphae, and I can think of several very prevalent diseases which use them such as oral candidiasis. Why, then, is thrush so hard to treat? If we could target the mechanisms by which hyphae are used, or prevent them from growing at all, why haven't drug companies exploited this fairly obvious difference between humans and fungi? Is it simply that there isn't a known drug which can disrupt hyphae activity without harming humans (due to a similar protein or such things)? I'm not a pharmacist or pharmacologist so there must be a decent reason why this has been excluded from drug companies checklists? Regards, --—Cyclonenim | Chat  19:06, 25 February 2010 (UTC)[reply]

To answer your specific question, hyphae are not clinically useful antifungal targets for several reasons. First, hyphae, if anything, act as protective structures that insulate the fungal cell to a certain degree (similar to the bacterial capsule of certain species such as the one that causes gonnorrhea), and the fungus is perfectly able to get along without it. I should also note that most fungi that parisitize humans do not grow as molds. Candidiasis is caused by candida albicans, which grows as a yeast. Yeasts by definition do not grow as hyphae. Currently available antifungals do exploit differences between human and fungal cells. Most antifungals work by inhibiting the production of a molecule incorporated into fungal cell walls, called ergosterol. Some have other mechanisms, though the cell wall is a handy target being that it is chemically very different from human cell membranes. It is true that there is a small minority of fungi that causes some degree of human grief, however I will remind you that, in addition to being very useful in our food supply, a fungus gave us the first clinically useful antibiotic: penicillin. Tuckerekcut (talk) 02:54, 26 February 2010 (UTC)[reply]
Despite what you might infer from some Novartis advertisements, drug companies are motivated by revenue. They thoroughly enjoy marketing drugs that treat (but do not cure) widespread, chronic, perhaps mildly degenerative conditions that are at least somewhat life-threatening. This way, many patients will stick to their drug regimens for the remainder of their lifetime. Oral candidiasis does not fit well into this category because of low morbidity and mortality related to infection and relatively low incidence (such as in comparison to diabetes). Moreover, current forms of medication (such as clotrimazole troches and various other gels and rinses) are pretty good at reducing fungal levels to normal, allowing bacterial oral flora to re-establish their usually more prominent levels -- so why spend time, money and effort to try to develop better medications? DRosenbach (Talk | Contribs) 05:10, 26 February 2010 (UTC)[reply]
Fascinating, thanks. Though I've heard that candidiasis will effect 75% of women once, 50% twice and 25% reoccuringly, though I don't have a statistic to back that up, just heard it in a lecture yesterday. If someone found a cure for thrush, they could make billions of it straight away and then continue to make money as newborns will still get it. Regards, --—Cyclonenim | Chat  10:23, 26 February 2010 (UTC)[reply]
In the United States, the unified move to eradicate the widespread prevalence of syphilis was generated not by any of the terrible sequelae of the disease except for it leading to birth defects and/or spontaneous abortion. It was only because of this maternal-fetal public health initiative that the CDC and/or other disease prevention coalitions pushed for curbing the disease. Appreciation that the signs and symptoms of syphilis (which, by today's standards are incredibly horrifying) did nothing to really create a great worry of morbidity, one can then see why the sequelae of candidiasis (even by today's standards) do not generate greater concern (and excluding young infants who contract thrush primarily because of their still-undeveloped immune system, it would mainly be found among the HIV+, who, as a cohort, are often less disturbed by their oral health, either due to other more pressing systemic health issues or because of indigence. DRosenbach (Talk | Contribs) 18:34, 26 February 2010 (UTC)[reply]

What is A and what is AH2?

There are several articles with the generic format:

In enzymology, a Delta12-fatty acid dehydrogenase (EC 1.14.99.33) is an enzyme that catalyzes the chemical reaction
linoleate + AH2 + O2 crepenynate + A + H2O
The 3 substrates of this enzyme are linoleate, AH2, and O2, whereas its 3 products are crepenynate, A, and H2O.

Notice that those articles (listed below) link to A and AH2, which are not chemical substances!

The articles, as far as my search could go, are:

What is A and what is AH2? Albmont (talk) 20:08, 25 February 2010 (UTC)[reply]

I've seen AH2 applied to enthalpy change (no idea why) and to ascorbic acid (see here). Regards, --—Cyclonenim | Chat  20:58, 25 February 2010 (UTC)[reply]
Adenine. It looks like the usual cofactor for this class of enzymes is NADH or NADPH, which acts as a hydrogen donor to form NAD(P)+ as byproduct. DMacks (talk) 20:58, 25 February 2010 (UTC)[reply]
That'd probably make more sense than my suggestions... :) Regards, --—Cyclonenim | Chat  20:59, 25 February 2010 (UTC)[reply]
Ascorbate is a reasonable possibility for a redox-active item as you found. I searched for these specific enzymes (using the links in the infobox for the first example), which gave me a slightly more explicit reaction. Man, biochem is hard enough without having a pile of undefined and mis-wikilinked terms unless the reaction really does take place on a highway in Asia. DMacks (talk) 21:03, 25 February 2010 (UTC)[reply]
I traced the KEGG entries that mention "AH2", and they all appear to point to just a generic type (based on reactivity) rather than a specific compound (different enzymes work with different structures perhaps?). AH2: "Reduced acceptor; Hydrogen-donor". DMacks (talk) 20:16, 26 February 2010 (UTC)[reply]

Are we all immortal?

Hear me out on this. When you are born you can first be observed by the universe. When you die you no longer can be observed. However relativity only permits information to travel at the speed of light. Therefore (assuming the universe is infinite) there's a wavefront heading away from you at the speed of light from the moment your born and it only ends when you die. Yet that wavefront will propagate forever. A wavefront that contains all the information that is you. A shell maybe 80 light years thick forever expanding out across the universe. Anywhere in that shell you will always exists. TheFutureAwaits (talk) 20:33, 25 February 2010 (UTC)[reply]

No, I think you would not exist, only the photons bouncing off of you would exist, or perhaps a radio signal or similar with your voice on it. Googlemeister (talk) 20:37, 25 February 2010 (UTC)[reply]
See that initially seems like the easy answer. But consider to an observer hundreds of lightyears away I won't exist until that light reaches them. TheFutureAwaits (talk) 20:41, 25 February 2010 (UTC)[reply]
Your image is not you--it's neither the complete information content of you nor the actual embodiment of that info. There's some plausibility to the idea "if there was a completely precise and accurate description of you, you could be recreated exactly as you are using that dataset". The photons bouncing off you are not that complete dataset (although they are influenced by some/much of it beyond the skin surface). DMacks (talk) 20:46, 25 February 2010 (UTC)[reply]
This seems like it's just a semantic argument about what it means to exist at a particular time and place. If you want to define existing at a particular time as still having an impact on events, then sure you exist forever everywhere inside of your future light cone. Is that a useful way to define it? Rckrone (talk) 20:50, 25 February 2010 (UTC)[reply]
From any given point of view you will exist for only a finite period of time.
More to the point (I think), you will only ever being observed having a finite number of interactions with the world around you. If you don't learn to ride a bicycle now in your (locally observed) life time, no one in the Andromeda galaxy will ever observe you riding a bicycle. APL (talk) 20:55, 25 February 2010 (UTC)[reply]
True, you can't change things but to an outside observer you will be making those decisions in realtime. TheFutureAwaits (talk) 20:59, 25 February 2010 (UTC)[reply]
When you talk about your existence do you mean just the image of your warm body that a camera can capture? If so then by your understanding "you" are indeed an expanding shell. The catch is that some of us think we are not just images. You can't see me so do you think I exist? Cuddlyable3 (talk) 20:57, 25 February 2010 (UTC)[reply]
We aren't just images but we ARE all information. Your actions can only have an impact within their light cone. So until a place intersects that lightcone you won't exist. TheFutureAwaits (talk) 21:01, 25 February 2010 (UTC)[reply]
(after ec) "A wavefront that contains all the information that is you." Ah, but that's not you. That's just a total encyclopaedian record of you, a dead thing, unable to act by itself and unconscious of itself. (I'm always fond of an intellectual discussion where I postulate that a candle flame is alive and squash away those counter arguments like not having a soul by asking what would be a proof for anything/anyone to have a soul.) 95.115.141.196 (talk) 20:58, 25 February 2010 (UTC)[reply]
Only the soulless need that truth proven. Cuddlyable3 (talk) 21:13, 25 February 2010 (UTC)[reply]
Quantum immortality is far more interesting. --Tango (talk) 21:17, 25 February 2010 (UTC)[reply]
I think the OP is reaching for something more holistic (along the lines that each of your movement has a subtle gravitational impact on everything in the universe...). arguably true, and karmically interesting, but hardly constituent of any realistic form of immortality. --Ludwigs2 21:30, 25 February 2010 (UTC)[reply]
I'd rather agree if you'd say we are all eternal (leaving aside all possible as well as unprovable religious metaphysics). 95.115.141.196 (talk) 21:45, 25 February 2010 (UTC)[reply]
Well, there's provability, and then there's truth. Which are you really more interested in — whether it can be proved that your consciousness can exist forever, or whether your consciousness will in fact exist forever?
If you're genuinely more interested in whether it can be proved, then fine, you can probably ignore the question. But I doubt that's what most people really want to know. -Trovatore (talk) 21:54, 25 February 2010 (UTC)[reply]
I'm really more interested in provability, as an intellectual exercise, for I know about truth already. 95.115.141.196 (talk) 21:58, 25 February 2010 (UTC)[reply]

There is a whiff of arrogance hopefulness about assuming immortality from a mechanical argument. The OP's prized shell of existence gets tenuously thin walled as it expands. Given the size of the Universe our OP gets stretched down to something thinner than a balloon, then down to a membrane of Nanoparticles and eventually down to subatomic particles that are unknowably adrift in the background radiation of the Universe that certainly won't stop expanding just so the OP can catch up. Cuddlyable3 (talk) 22:22, 25 February 2010 (UTC)[reply]

Doesn't one continue to exist after death even if their body is in a mausoleum or some such place? Bus stop (talk) 22:31, 25 February 2010 (UTC)[reply]
The wall would not get thinner. Let's say you lived 80 years. The shell would be 80 light years thick, always. The physical volume of space it contained would grow but the thickness would be the same.TheFutureAwaits (talk) 23:14, 25 February 2010 (UTC)[reply]
actually, that's not true - relativistic effects would ensure that the width of the "wall" would expand/contract/bend in interaction with gravitational wells... just saying...--Ludwigs2 23:20, 25 February 2010 (UTC)[reply]
You've got the right idea, but a bit backwards. The first fact is that you exist -- the universe is only tertiary to your consciousness. When you dream, your mind is imagining things, but those imaginings mean something, while all the goings-on outside your mind are totally irrelevant until you awake. Anyway we may be all theoretically immortal, but when you die, you lose all your memories, which is not preferred! So it's a yes-but-no kinda answer. Vranak (talk) 00:24, 26 February 2010 (UTC)[reply]
When you are born you can first be observed by the universe. When you die you no longer can be observed. You existed before birth, as a fetus, an embryo, a zygote--which existed before as egg and sperm, and so on. You exist after death, as a corpse, as physical matter. We aren't just images but we ARE all information. Your actions can only have an impact within their light cone. So until a place intersects that lightcone you won't exist. Physical matter is information just the same, interacting with its environment, changing, having an effect and being effected, being "observed by the universe". Why must "your actions" only begin with birth and end with death? Are "your actions" only those done with conscious attention and intent? You digest food without conscious attention and intent--it just happens. After death your body decomposes without conscious attention and intent. I don't see the difference in terms of being "observed by the universe" or "information about you" or "actions" that "have an impact". Perhaps the question to ask is not "are we immortal?" but "what is it that lives and dies?" Pfly (talk) 11:17, 26 February 2010 (UTC)[reply]
Perhaps the question could be rephrased : "Are we radiating enough information, brain waves, gravity, etc, that the information exists, however inaccessible, to reconstruct the information that constitutes our minds."
If true, then I suppose the answer to the question would be "Yes, but not in a useful way unless some benevolent god-like alien decides to reconstitute you." APL (talk) 16:20, 26 February 2010 (UTC)[reply]
The uncertainty principle may get in the way there. --Tango (talk) 18:22, 26 February 2010 (UTC)[reply]

Tallow

where can i buy Tallow or Suet ?

A decent supermarket? --Tango (talk) 21:23, 25 February 2010 (UTC)[reply]
Suet you can get at any birding store - it's a common element of winter birdfeed. tallow is odder - it only has industrial uses in the modern world, and I don't know of anyplace to get it as a commercial product. if there's a rendering plant in your vicinity, you might give them a call and ask. or you can probably shop for it online. --Ludwigs2 22:12, 25 February 2010 (UTC)[reply]
Tallow is (was) used for caulking in wooden boats. Try asking at a chandler. CS Miller (talk) 22:20, 25 February 2010 (UTC)[reply]
In the UK you can find suet in most supermarkets, both beef suet and vegetarian 'suet'. Look by the baking supplies. As CS Miller says, tallow will be more difficult. 86.177.121.239 (talk) 22:23, 25 February 2010 (UTC)[reply]

I'd just like to caution you that bird-grade suet and industrial-grade tallow aren't made with human hygiene in mind, so don't eat them. If they were made hygienically then they'd be safe (they are just purified fats), but probably wouldn't taste that great. Should be fine for candle making tho. Tallow was used for candles, not sure about suet. CS Miller (talk) 22:38, 25 February 2010 (UTC)[reply]

I think suet can be used for candles as well (it's not that different from tallow) but I suspect it would smoke and stink badly... --Ludwigs2 23:05, 25 February 2010 (UTC)[reply]
Like most things, you can buy tallow on eBay. Or try Google products search: [6].
As general advice, if you want a better answer to this sort of question you should specify (a) where you are (b) what sort of tallow and suet you want, e.g. for what application (c) how much you want to buy and (d) whether you're looking for online or real-world shops. --Normansmithy (talk) 18:00, 26 February 2010 (UTC)[reply]
If you want tallow, you will probably have to make it, by rendering beef kidney fat, which you can probably obtain cheaply from a butcher by asking them to put some aside for you. It isn't very hard to do -- I've done it myself in an experiment at making old-fashioned pemmican. Looie496 (talk) 18:53, 26 February 2010 (UTC)[reply]

Is this Edge Peak?

Hello, I live in Langley, BC. I saw this mountain to the North. Is this Edge Peak? --The High Fin Sperm Whale 21:06, 25 February 2010 (UTC)[reply]

This shot, from a slightly different angle, is still clearly the same mountain and is called Mount Robie Reid, and that article is in need of a photo. Mikenorton (talk) 21:43, 25 February 2010 (UTC)[reply]
Messing around with Google Earth, if I approach Mount Robie Reid (using the article's coordinates) from the south-southwest at a suitably low elevation, I can get a view that looks convincingly like your photo. Mikenorton (talk) 23:10, 25 February 2010 (UTC)[reply]
Edge Peak lies North-East of Langley. Does the OP's picture fit the profile? Cuddlyable3 (talk) 22:39, 25 February 2010 (UTC)[reply]
Thanks a lot guys, that would be it! --The High Fin Sperm Whale 03:50, 26 February 2010 (UTC)[reply]

Fog

Does fog occur worldwide? Thanks in advance. Rimush (talk) 23:12, 25 February 2010 (UTC)[reply]

I would think so, and there's nothing in our article on Fog to suggest that it doesn't. —Steve Summit (talk) 23:25, 25 February 2010 (UTC)[reply]
all fog requires is sufficiently high humidity, a temperature below the dew point and above freezing, and condensation nuclei. I can't think of anyplace that would preclude those conditions (except possibly in the heart of a dessert, if there is a dessert where humidity is always negligible). --Ludwigs2 23:35, 25 February 2010 (UTC)[reply]
As our article states, some deserts are in fact quite well known for their dense fogs. Some animals are keenly adapted to that phenomenon, indicating that, in some deserts at least, it's a very regular thing. Matt Deres (talk) 04:25, 26 February 2010 (UTC)[reply]
It occurs with varying frequency, but it can occur anywhere. Some deserts may get fog only once every 10,000 years, or something, though. --Tango (talk) 00:42, 26 February 2010 (UTC)[reply]
According to this info, the highest recorded temp at the south pole is 7 degrees Fahrenheit. It would have to get quite a bit warmer to permit fog. Looie496 (talk) 02:04, 26 February 2010 (UTC)[reply]
You don't have to have liquid water for fog: ice fog. (As well, even with sub-freezing temperatures you can still have supercooled liquid droplets — small water droplets can remain liquid down to thirty or even forty degrees below zero.) TenOfAllTrades(talk) 04:16, 26 February 2010 (UTC)[reply]

February 26

Turning a switch backwards

I don't know how standard these types of switches are around the world, so I'll just preface this by saying that I'm in the US.

What happens if I turn the twist type switch, the one up near the light bulb, on my lamp backwards? Does the little knob (similar to a watch crown) just unscrew? Since all my lamps work, I don't want to risk breaking one by testing this out. And I'm certainly not going to take one apart. Thanks, Dismas|(talk) 04:30, 26 February 2010 (UTC)[reply]

I doubt there is any standard for the construction of twisting switches. It could turn freely, doing nothing. It could unscrew. It could refuse to move until you use enough force to break it. I expect there are switches out there that do each. --Tango (talk) 04:33, 26 February 2010 (UTC)[reply]
As Tango said... there are many types of twist switches. For any switch, there are many types. When I recently replaced a pull-chain switch, I was surprised to find four completely different types of pull-chain switches to choose from. -- kainaw 05:17, 26 February 2010 (UTC)[reply]

I have accidentally turned a twist switch backwards. The knob unscrewed from the switch (and returned to normal when I twisted it forwards again). As the last person said, this may not be the same for all of them. --Anonymous, 11:52 UTC, February 26, 2010.

I've seen them that work both ways. I have a table lamp that unscrews and an old standard lamp which allows you to rotate the knob in either direction to turn the light on or off. SteveBaker (talk) 14:07, 26 February 2010 (UTC)[reply]

Lorentz transformations

Why do Lorentz transformations have to be linear? My teacher mentioned something about one-to-one correspondence (whatever that means), but could someone be more specific? Thanks. 173.179.59.66 (talk) 07:00, 26 February 2010 (UTC)[reply]

We have an article on One-to-one correspondence (actually under the more formal title of "Bijection" but the two terms mean the same thing, and the former redirects to the latter). --Jayron32 07:04, 26 February 2010 (UTC)[reply]
Alright, so why is one-to-one correspondence important, and why does linearity conserve it? —Preceding unsigned comment added by 173.179.59.66 (talk) 07:48, 26 February 2010 (UTC)[reply]
One-to-one correspondence is important because it is desirable for the Lorentz transforms to express some sort of group structure. Without one-to-one correspondence, there would exist transforms without a well-defined inverse transform. It is desirable to have a group structure because then the transforms are universal - they're the same (structurally) for every reference frame and can transform from any reference frame to any reference frame. A linear function is an "easy" way to obtain a one-to-one correspondence that has an inverse map of the same form. It may even be the only way (perhaps a proof could start with the inverse function theorem). 118.209.163.175 (talk) 08:07, 26 February 2010 (UTC)[reply]
Unfortunately, my knowledge of linear mapping is pretty non-existant. Is there a more physics-oriented explanation? —Preceding unsigned comment added by 173.179.59.66 (talk) 08:50, 26 February 2010 (UTC)[reply]
I think you should tell your physics teacher that you have no idea about what one-to-one correspondence means. I'm sure he will have a long chat with your math teacher. DVdm (talk) 08:55, 26 February 2010 (UTC)[reply]
There are plenty of one-to-one functions that are non-linear. For example, . The Lorentz transformation has to be linear to ensure that light travels in straight lines (in a flat space-time) for all inertial observers. Gandalf61 (talk) 11:31, 26 February 2010 (UTC)[reply]
Ok, so how does non-linearity make light not travel in straight lines in other reference frames? —Preceding unsigned comment added by 173.179.59.66 (talk) 17:00, 26 February 2010 (UTC)[reply]
Snell law. Any inhomogeneity of the medium may lead to the bending of the path of the lightray. Dauto (talk) 19:02, 26 February 2010 (UTC)[reply]
The real reason the Lorentz transformations must be linear is that we require them to respect space-time homogeneity. As gandalf hinted above, the fact that light travels in straight lines is strong experimental evidence for space-time homogeneity. Dauto (talk) 16:49, 26 February 2010 (UTC)[reply]

Mistake in Textbook?

I believe I found a mistake in my physics textbook, and Introduction to Mechanics by Kleppner/Kolenkow. I wanted to confirm that my assessment is correct.

Example 7.3 in Chapter 7 goes as follows:

Consider a particle rotating in a verticle plane as shown in the sketch. [The sketch consists of a three-dimensional Cartesian coordinate system with a circle representing the particle's plane of rotation. This plane is perpendicular to the xy plane, with the ω vector making a 45° angle with the x- and y- axes. The position vector of the particle, r, is said to be at an angle θ from the z-axis, with θ being in the clockwise direction]

First we shall calculate v directly from the relationship v = dr/dt. To find r, note that z = r cosθ, x = -r sinθ/√2, and y = r sinθ/√2. Hence,

Differentiating, we have, since r = constant,

Next, we shall find the velocity from v = ω × r. Assuming that ω can be resolved into components,

we have

as expected.

However, unless I've made a mistake in evaluating the cross product, I get the negative of their answer, so there does appear to be a discrepancy! Is this a mistake on my part, or because they defined positive θ to be in the clockwise direction (and dθ/dt would be one, which was tacitly assumed in this example but should be -1 due to how they defined θ)? —Preceding unsigned comment added by 173.179.59.66 (talk) 07:47, 26 February 2010 (UTC)[reply]

Yes, there is a sign mistake. I don't know if it is in the expression for , , or . Could be any one of them. I would have to see the picture in order to tell which one is wrong. Note that . Dauto (talk) 16:20, 26 February 2010 (UTC)[reply]

difference between distance and displacement

What is difference between distance and displacement —Preceding unsigned comment added by NADEEM BHAT (talkcontribs) 08:22, 26 February 2010 (UTC)[reply]

Distance is the absolute value of displacement. When you go from a place with coordinate x=4 to a place with coordinate x=1, your displacement is -3 and you covered a distance 3. See our articles Distance and Displacement. - DVdm (talk) 08:47, 26 February 2010 (UTC)[reply]
I would say distance is the norm of displacement. If you are working in one dimension, then that's the same thing as absolute value, but with higher dimensions displacement is a vector. --Tango (talk) 18:01, 26 February 2010 (UTC)[reply]

Countering momentum

Suppose you are playing football (association football or soccer if you will) and you receive pretty strong pass from player exactly in front of you. If you take a shot without stopping the ball (back towards the player that sent the pass), the speed/strength of the shot feels substantially more powerful then if ball was resting. Obviously, the added strength comes from kinetic energy other player invested, but what is the exact mechanism behind this? To me, logical thing is that you would spend part of kinetic energy you invest on actually countering other player's kinetic energy, thus stopping the ball before sending it back, resulting in weaker shot overall, but opposite is true. Can you shed some light on this phenomena? --124.148.229.154 (talk) 08:57, 26 February 2010 (UTC)[reply]

Try Deflection (physics) and Collision --Digrpat (talk) 13:37, 26 February 2010 (UTC)[reply]
Some of the kinetic energy of the incoming ball gets turned into elastic energy as the ball is compressed/dented/squashed as it hits your foot. As the ball departs your toe, heading off in the opposite direction, the ball uncompresses - giving back that stored elastic energy as kinetic energy in the opposite direction. The ball is essentially acting like a spring. But because every action has an equal and opposite reaction, your foot feels that much higher impact. That's why balls have to be made of inflatable, rubbery kinds of things. SteveBaker (talk) 14:05, 26 February 2010 (UTC)[reply]
Your mistake was to assume that part of the invested energy is used to counteract the already present kinetic energy. That doesn't happen. In fact, the two add up. Dauto (talk) 16:03, 26 February 2010 (UTC)[reply]
Indeed. I suspect our OP is confusing "kinetic energy" and "momentum" - the two are not the same thing. Kinetic energy is mass times velocity squared. Momentum is mass times velocity. In a collision like this, both momentum and energy have to be conserved. This matters because momentum is a signed quantity - when the ball is coming towards you, its momentum is the opposite of what it is when it's moving away. For momentum to be conserved, your foot has to exchange some momentum with the ball. But kinetic energy is a scalar quantity - so it's perfectly OK that the energy from your foot going forwards adds to the energy of the ball coming towards you when it turns around and heads off across the pitch. The elasticity of the ball allows it to briefly store that kinetic energy as elastic energy while the ball slows down and transfers momentum to your foot. SteveBaker (talk) 21:19, 26 February 2010 (UTC)[reply]

fuel oil

what is hsd,fo,lshs? what are thire advantages and dissadvantages? is this all are commersially available fuels like petrol,diseal etc. —Preceding unsigned comment added by 117.204.85.94 (talk) 10:24, 26 February 2010 (UTC)[reply]

They could be many things. Could you give some of the context that they are being used in? --Phil Holmes (talk) 10:07, 26 February 2010 (UTC)[reply]
Seeing as you were quite vague I didn't have much to go on, but this is the science reference desk so that gives us a hint. Running a good search here for "LSHS" "FO" gave me a lot of results for two types of hydrocarbon in oil. Regards, --—Cyclonenim | Chat  10:27, 26 February 2010 (UTC)[reply]
This web page: "suppliers.jimtrade.com/91/90371/56222.htm" (which I can't link directly to because it's on the Wikipedia blacklist for some reason) compares LSHS to FO. This article is about High Speed Diesel. SteveBaker (talk) 13:55, 26 February 2010 (UTC)[reply]

Animal tastebuds

Hi

1. What animal's tastebuds are similar to a human being's? 2. What type of animal's tastebuds are more complex or has a better sense of taste then a human being's

Thanks, NirocFX

41.193.16.234 (talk) 11:14, 26 February 2010 (UTC)[reply]

I believe that taste is a matter of physiology. Cats for instance have little use for sweetness, so they are not going to go crazy for ice cream or honey if presented with it. So there's not really a more 'complex' set of tastes -- organisms have evolved faculties in response to what's available in their environment. Vranak (talk) 17:35, 26 February 2010 (UTC)[reply]
I believe you might be confusing, as most do, the gustatory (taste) and olfactory (smell) senses. Taste consists of merely five basic sensors, while olfaction provides a complete spectrum of virtually limitless combinations of aromas and odors. So while the evolutionary benefit of taste is a more protective one (in that bitter/sour tastes may warn for dangerous alkaline/acidic agents, and sweet tastes are generally harmless (except, of course, for ethylene glycol), and salty is tied to a drive for Na+ intake), olfaction is a sort of everything sense, in that odors can represent dangerous predators, tasty prey, cute potential mating partners, etc. So while taste is probably similar across the board, I would say that the vast majority of mammals possess a greater sense of olfaction than do humans. If I'm off track, and you really meant to focus on gustation...I don't think there can be anything more complex than the 5 taste sensations we currently know about. DRosenbach (Talk | Contribs) 18:44, 26 February 2010 (UTC)[reply]
The human sense of smell is actually vastly underrated. Partly because much of what we attribute to "taste" is in fact "smell" - but also because we don't use that sense very much. There was an experiment done about a year or two ago to see if humans could follow a scent trail such as a dog can. While it's clear that we're not up there with the bloodhounds, even completely untrained, unpracticed, blindfolded humans did pretty well at following a trail. SteveBaker (talk) 21:10, 26 February 2010 (UTC)[reply]

NE Snowstorm

In this, it appears that this weather system looks like a hurricane cloud. So according to this map, the eye of the storm is over Hempstead LI. Does that mean the right now, Hempstead sees clear skies? --Reticuli88 (talk) 15:29, 26 February 2010 (UTC)[reply]

Yes.Accdude92 (talk to me!) 16:00, 26 February 2010 (UTC)[reply]
Not really. Extratropical cyclones don't have eyes. Dauto (talk) 16:30, 26 February 2010 (UTC)[reply]

Using mouthwash and brushing teeth

Is it worth using mouthwash in addition to brushing your teeth? Is also using mouthwash likely to lead to any significantly less need for dental work over the years? Thanks 78.146.242.196 (talk) 16:27, 26 February 2010 (UTC)[reply]

Yes, have you read our mouthwash article?--Shantavira|feed me 16:47, 26 February 2010 (UTC)[reply]
Have you read the article? Because if you had, your answer would be "No". 89.242.83.202 (talk) 18:55, 26 February 2010 (UTC)[reply]
It depends. Really you have to ask yourself, does mouthwash make your mouth feel cleaner? Or do you find it harsh? It certainly has its uses, and its limitations. Vranak (talk) 17:32, 26 February 2010 (UTC)[reply]
Mouthwash certainly provides a statistically significant benefit, but whether or not that translates to a clinical benefit can be debated. To begin, one must first discriminate between various mouthwashes -- those that are antiseptics (such as povidone iodine, chlorhexidine, Listerine, Crest Pro-Health), those that are breath fresheners (such as Scope) and those that are meant to provide other benefits, such as fluoride rinses (such as Act). So assuming we are discussing antiseptics, mouthwashes will not kill all bacteria because there is no way to get the mouthwashes of enough antiseptic concentration to remain in contact with oral pathogens for a long enough time -- even 2% chlorhexidine requires 10 minutes to kill P. gingivalis, a serious periodontal pathogen. And periodontal bacteria are generally found in the gingival sulcus/periodontal pocket and mouthwashes cannot get into these areas. And even if they are forced under the gumline with a Waterpik or similar apparatus, gingival crevicular fluid flow rates completely renew the pocket fluid 40 times an hour, giving any drug placed under the gumline about a 1 minute half-life. That being said, 'dilution is the solution to pollution' -- and removing bacteria from oral surfaces (enamel and gums and other soft tissue mucosa) certainly aids in preventing translocation of bacteria from, say, the tongue or the tonsils back onto teeth after brushing (as no one brushed tonsils and no one brushes the underside of the tongue or far back enough on the tongue to reach all the bacteria). So yes -- proper use of good antiseptic mouthwashes certainly contribute to dental health. DRosenbach (Talk | Contribs) 19:00, 26 February 2010 (UTC)[reply]
Not wanting to spread disinformation, so shoot me down immediately if it's all scare stories - but don't newspapers come out with a link between mouthwashes and oral cancers once every two years or so? Vimescarrot (talk) 21:46, 26 February 2010 (UTC)[reply]
See Listerine#Safety. Vranak (talk) 21:56, 26 February 2010 (UTC)[reply]

Sand grains on earth

The site http://www.wolframalpha.com/ gives the number of sand grains on earth as (1x10^20 to 1x10^24). I want to know how such a number is obtained. What would be the procedure and the "size" of error involved. Thanks.--119.155.13.92 (talk) 16:33, 26 February 2010 (UTC)[reply]

Well, one thing I see right off the bat is that they give a range that is 4 orders of magnitude. That would be like estimating that there are somewhere between 60 million and 600 billion people on earth, so their procedure results in a substantial error size... Googlemeister (talk) 16:39, 26 February 2010 (UTC)[reply]
You can get an estimate by working out the total area covered by sand (A), the average depth of the sand (D), the average volume of a sand grain (V) and the average packing efficiency of sand (E). The number of sand grains is then A*D*E/V. There is going to be a fairly large error on each of those factors so the final value has a very high error (+/- 2 orders of magnitude in the figures you give). --Tango (talk) 18:06, 26 February 2010 (UTC)[reply]
Googling gives quite a lot of information on how you might calculate this[7][8][9][10]. Basically how many grains in a unit volume, times total volume of sand. --Normansmithy (talk) 18:07, 26 February 2010 (UTC)[reply]

How long do preserved animals "keep"?

Shortly before I took high school biology, my teacher purchased a collection of several preserved animals for dissection. I was talking with my teacher earlier today and learned that a crawfish and a foetal pig from this collection are still in one piece. Since they've been stored in a cupboard without temperature control, are they likely still to be good for dissection purposes? I can't remember for sure, but I'm guessing that the biology class was seven years ago. Nyttend (talk) 16:59, 26 February 2010 (UTC)[reply]

As long as the specimens are covered by the fixing solution (often formalin and some other goodies), they should be fine still. The fixing solution prevents bacterial and fungal growth, either of which might destroy the specimens. Also, once the tissues are fixed, they don't get more fixed with time so there shouldn't be any change in the tissue over the years. The original specimens used to draw Grant's Atlas of Anatomy (first ed. is 1943) are still on display (and in good condition) at the University of Toronto's Anatomy department. -- Flyguy649 talk 18:12, 26 February 2010 (UTC)[reply]

Circular logic? How was Avogadro's number originally determined?

I looked at Avogadro constant and the link at footnote 7 about Loschmidt doesn't work. The methods described in the body of the article all seem to be rather circular. The Coulometry method uses Faraday's constant, which is the charge of a mole of electrons. You can't know Faraday's constant then before you even know how many a mole of something is. The electron mass method which is next in the article uses the molar mass of the electron, once again it seems circular. And then the X-ray crystal density method uses the molar volume. How can you know the volume of a mole of things before you know how many a mole is? So where is the description of an experimental procedure to determine the Avogadro number that doesn't require knowing how much a mole of something is beforehand? 20.137.18.50 (talk) 17:19, 26 February 2010 (UTC)[reply]

I've always wondered this myself... Nyttend (talk) 17:34, 26 February 2010 (UTC)[reply]
Didn't you read the article you linked? It explains how to directly measure the Faraday's constant before you know how much a mole is. Let us know if you need more explanation about how it worked. Dauto (talk) 18:09, 26 February 2010 (UTC)[reply]
I followed the link to Faraday constant which said F was first determined by weighing the amount of silver deposited in an electrochemical reaction in which a measured current was passed for a measured time, and using Faraday's law of electrolysis. But what is current measured in? Coulombs per second. What's a Coulomb? The charge of a mole of electrons. Sorry, still I don't see the part where F gets determined without knowing anything about moles. 20.137.18.50 (talk) 18:38, 26 February 2010 (UTC)[reply]
Yes, you are right. This method only works if you have an independent measurement of the charge of the electron which they had. See the millikan droplet experiment. Dauto (talk) 18:51, 26 February 2010 (UTC)[reply]
Thanks20.137.18.50 (talk) 19:10, 26 February 2010 (UTC)[reply]
A coulomb is only about moles of electrons. --Tardis (talk) 23:38, 26 February 2010 (UTC)[reply]
According to Scientific American "The first person to estimate the actual number of particles in a given amount of a substance was Josef Loschmidt, an Austrian high school teacher who later became a professor at the University of Vienna. In 1865 Loschmidt used kinetic molecular theory to estimate the number of particles in one cubic centimeter of gas at standard conditions." Avogadro didn't actually calculate his number; it wasn't calculated till 1909.[11] --Normansmithy (talk) 18:11, 26 February 2010 (UTC)[reply]
Sorry, that's exactly what it says in the Wikipedia article. Which reiterates Dauto's point. --Normansmithy (talk) 18:12, 26 February 2010 (UTC)[reply]

Geochemical/Mineralogical Equilibrium

Can this combination exist in equilibrium? Why/why not?

Calcite+Dolomite+Enstatite+Wollastonite+Diopside

I believe I have decided yes, but I'm not quite sure how to back my answer. —Preceding unsigned comment added by 131.123.82.201 (talk) 17:37, 26 February 2010 (UTC)[reply]

OK, dragging my mind back a few years (actually that's decades now, 'shudders'), to when I did metamorphic petrology and specifically the calc-silicates, I would say that in the case of a metamorphosed limestone with dolomite, there is likely not enough quartz to react with all the carbonates, so that calcite and dolomite will remain (note that they are only in equilibrium if there's no quartz available). That leaves wollastonite, the product of the reaction calcite + quartz and diopside from the reaction of quartz with dolomite. Not so sure about where the enstatite is coming from though, normally a high temperature and pressure mineral. Mikenorton (talk) 19:18, 26 February 2010 (UTC)[reply]


Could we also determine that if the combination of phases coexist at some given temperature, would this also be true if the temperature were to decrease by a few degrees? —Preceding unsigned comment added by 131.123.82.201 (talk) 23:28, 26 February 2010 (UTC)[reply]

Becoming an engineer in the UK with a Msc.

If you have a degree in a natural science, can you become an engineer in the UK with a Master of sciences? ProteanEd (talk) 18:47, 26 February 2010 (UTC)[reply]

This document explains the requirements for the various levels of professional accreditation for engineers in the UK. To become a Chartered Engineer, you need a Bachelors and Masters, both in Engineering or another recognised subject. A general science degree isn't sufficient, apparently. --Tango (talk) 21:04, 26 February 2010 (UTC)[reply]

Radiometric dating of dinosaur fossils

After reading Radiometric dating and Dinosaur, I don't really understand how dinosaur fossils are dated. (We have no specific dinosaur fossil article.) Is the age of the fossil inferred from radiometric dating of the rock that it's embedded in? Which radiometric dating technique is used? If I understand correctly, the mineralization of a dinosaur bone does not sound like an event whose date can be determined. Comet Tuttle (talk) 19:08, 26 February 2010 (UTC)[reply]

My ballpark understanding was that they measure the amount of Nitrogen-14 to Carbon-14 in the fossil, knowing has a half life of 5730 years. I could be wrong on some details of what I just said.20.137.18.50 (talk) 19:15, 26 February 2010 (UTC)[reply]
I don't think carbon dating can be used. All the bone is replaced with minerals, and I believe the fossils are too old for carbon dating to be effective, anyway. Comet Tuttle (talk) 19:23, 26 February 2010 (UTC)[reply]
(EC) Radiocarbon dating only works for things that lived within ~60000 years. For dinosaurs, one of the other methods described in Radiometric dating would be required, although I'm not sure which one it is. -- Flyguy649 talk 19:26, 26 February 2010 (UTC)[reply]
This answer suggests the rocks above and below the fossil are dated to set a bracket for the age of the fossil. The isotopes used are uranium-238, uranium-235 and potassium-40. -- Flyguy649 talk 19:32, 26 February 2010 (UTC)[reply]
I'm still not understanding something here. I understand carbon dating, but that's out. I'm confused as to what exactly is the date that's measured by the other methods described in Radiometric dating. The potassium-argon dating article clearly states that it measures when the liquid rock solidifies, and I assume that Uranium-uranium dating and Uranium-lead dating measure the same thing. (Those two articles should be clearer for the sake of laymen like myself. For all I know, the dating methods are measuring how long it has been since the uranium was ejected from a supernova.) If I'm correct and these are the types of techniques being used, then dating the dinosaur fossils seems really inaccurate. All that's being measured is a layer of nearby molten rock that solidified, which doesn't necessarily correspond to the eon that the dinosaur lived; all you might get, it seems to me, is a maximum age (from the layer beneath the fossil) and a minimum age (from any layer on top of the fossil, if the fossil was lucky enough to have magma run on top of its rock layer without destroying the fossil). Comet Tuttle (talk) 20:09, 26 February 2010 (UTC)[reply]
The supernova idea is completely off the mark. The Potassium Argon Method measures the time since the formation of the rock. You are right that what one gets is a maximum and a minimun age but that can be enough to give an accurate estimate for the age of the fossil in most cases. There is no need for lave to run over the fossil. Ashes from volcanic eruptions can be used and fortunately large volcanic eruptions that leave an world wide imprint on rocks happen often enough for the method to ve quite accurate. Dauto (talk) 20:22, 26 February 2010 (UTC)[reply]
What we're dealing with here is chronostratigraphy, which basically combines biostratigraphy (ages based on fossil assemblages - if you have lots of different types of 'bugs' you can often be very precise) with geochronology, based on radiometric dating. The International Commission on Stratigraphy works out type sections of each subdivision of the geological time scale, complete with 'golden spike' to mark the boundaries. Once you have pretty much all the Phanerozoic (since the start of the Cambrian) sedimentary rocks dated in this way, then you need to tie it to an absolute time scale, which is where all the various radiometric dating techniques come in. You then need to search the world for some place where your rock unit contains lava flows, that are clearly interbedded with the sediments, take samples and date them and then eventually you have a time scale that allows you to say "this dinosaur lived 84 million years ago" or whatever. Mikenorton (talk) 20:49, 26 February 2010 (UTC)[reply]
Of course, the two work hand in hand, nobody actually waits for the ICS before they go out to date lavas The biostrat and the geochronology are both being constantly refined, which can mean major geological boundaries occasionally shifting around by a few million years or more. This can be a bit perplexing if you've just come up with a model for the timing of an orogenic event based on a mix of radiometric ages and fossils, although generally the changes sort out more problems than they create. Mikenorton (talk) 21:10, 26 February 2010 (UTC)[reply]

euthanasia

where can i watch the euthanasia documentary "Mademoiselle and the Doctor" (ideally online) iv looked everywhere and cant find it —Preceding unsigned comment added by Thekiller35789 (talkcontribs) 19:20, 26 February 2010 (UTC)[reply]

Curling

Why would spinning a curling stone cause its trajectory to curve? And how does sweeping act to make the stone go farther? —Preceding unsigned comment added by 173.179.59.66 (talk) 20:11, 26 February 2010 (UTC)[reply]

The spinning is probably related to the Magnus effect. The sweeping melts a thin layer of ice, reducing friction. --Tango (talk) 20:16, 26 February 2010 (UTC)[reply]

Because curling stones have a three-dimensional lattice structure, so the resultant friction between them causes them to interact angularly. When two stones touch, it's more like velcro clasping, with real pull, than two pieces of paper rubbing together with little effect save for sound. Vranak (talk) 21:51, 26 February 2010 (UTC)[reply]

Putting spin on the stones does affect how they collide, but that's not what the OP was talking about. They also curve as they move on their own (like a curveball). --Tango (talk) 21:58, 26 February 2010 (UTC)[reply]

It's a bit simpler than all that. Frictional forces are speed-dependent, and when the curling stone is spinning while it slides, one side moves across the ice faster than the other side, and therefore encounters more friction. Looie496 (talk) 22:45, 26 February 2010 (UTC)[reply]

If I recall correctly, sliding friction is actually not very speed-dependent (quite unlike air resistance). That doesn't mean you're wrong ("not very" not being the same thing as "not at all") but I'd like to see some support for the claim.
Does anyone here actually have any experience with curling? Say, if the stone is rotating counter-clockwise, does it tend to bend left or right? --Trovatore (talk) 00:02, 27 February 2010 (UTC)[reply]
The thing that make curling's physics complicated is that the ice isn't kept smooth like a skating rink - before the match, it's sprinkled with water droplets that freeze and make a surface that's described as "pebbled". The stones are crazy heavy (they are solid granite) - and the contact area is tiny because it's riding on the top of those frozen droplets and the underside of the rocks are concave. With a lot of weight and a small contact area the pressure on the ice is huge. That pressure increase causes the ice to melt and allows the rock to slide on water - not ice. So the spinning must be having a hydrodynamic effect as well as "normal" friction. The spinning will cause all sorts of complex interactions with the thin liquid layer. The fact that pressing hard on the brushes has an effect on the path of the stone shows that water on the surface of the ice makes it more slippery and the amount of water make a huge difference to how the stone moves. SteveBaker (talk) 02:24, 27 February 2010 (UTC)[reply]
I curl, and when the rock is spinning clockwise, it curls to the right (from the point of view of the thrower).24.150.18.30 (talk) 04:09, 27 February 2010 (UTC)[reply]

FIDEL CASTRO TRAVELS TO THE USA

1960 doesn't seem like such a long time ago

fidel has come to the usa many times but there are no articles about the trips that i can find. can anyone give me a link or 2? —Preceding unsigned comment added by 67.246.254.35 (talk) 22:23, 26 February 2010 (UTC)[reply]

I'm not sure why you posted this to the science refdesk — the miscellaneous refdesk would be a better place.
But anyway — he has? You mean, since 1959? That strikes me as somewhat implausible. What is your reason for believing this? --Trovatore (talk) 23:57, 26 February 2010 (UTC)[reply]
Googling castro visit un shows he visited in 1960 (pictured), 1995, and 2000, at a minimum. Comet Tuttle (talk) 01:17, 27 February 2010 (UTC)[reply]
Oh, I see. Well, the UN is extraterritorial; it isn't US soil. I suppose he has to pass over US soil to get there. That's not exactly what I think of when I hear of him "coming to the USA", though. --Trovatore (talk) 01:48, 27 February 2010 (UTC)[reply]
The UN has an agreement with the US that it will allow people to enter the US when visiting the UN. I think they are usually restricted to NYC if they wouldn't otherwise have been allowed in. --Tango (talk) 02:52, 27 February 2010 (UTC)[reply]

Dark Energy not needed if gravitation converges to r instead of r^2 for large r ?!?

From the Dark energy article:

It is of interest to note that if the equation for gravity were to approach r instead of r2 at large, intergalactic distances, then the acceleration of the expansion of the universe becomes a mathematical artifact,[clarification needed] negating the need for the existence of Dark Energy.

I am a layman when it comes to physics of that kind, but that seems a bit too simple for not everybody to jump on it.

What would interest me, though, how would an alternative formula for large-scale gravitation look like? Is that where Einsteins cosmological constant comes in? —Preceding unsigned comment added by Xsansara (talkcontribs) 22:32, 26 February 2010 (UTC)[reply]

There are theoretical physicists investigating the possible reformulations of gravity that follow something other than an inverse square law at large distances, but they haven't got anything particularly convincing yet. --Tango (talk) 22:43, 26 February 2010 (UTC)[reply]
That sentence was added anonymously with no edit comment and no cited source (in 2008!). I removed it. -- BenRG (talk) 23:48, 26 February 2010 (UTC)[reply]
It wouldn't be hard to write down a function that falls off like 1/r² for small r and 1/r for large r – for example, 1 / (r ro tanh (r/ro)). It's easy to modify Newton's laws along those lines and see what happens—it wouldn't cause accelerating expansion, though. Modifying the force law in general relativity is much harder because in GR the 1/r² falloff isn't an independent assumption. It's an inevitable consequence of, roughly speaking, the fact that the surface area of a sphere is 4πr². You could try making one dimension of space loop around itself (making a cylinder) at some large scale; then at even larger scales space would effectively have one less dimension and the falloff might (I'm not sure) become 1/r. But that kind of looping has already been searched for and ruled out. The cosmological constant is broadly similar to making space hyperbolic instead of flat. In hyperbolic space, the surface area of a sphere is 4π r ro sinh (r/ro) 4π ro² sinh² (r/ro), which would give you a falloff law like 1 / (ro² sinh² (r/ro)), which is approximately 1/r² for r << ro but goes exponentially to zero for r >> ro. The cosmological constant actually goes repulsive for large enough distances, so this isn't a perfect analogy, but it is the right idea. -- BenRG (talk) 00:34, 27 February 2010 (UTC)[reply]

February 27